You are on page 1of 89
INTEGRATED SCHOOL PROGRAM 2020 - 21 CHEMISTRY Chemical Kinetics A WORD OF ADVICE Try 10 do the solved problems and exercises given, after completion of related topics in the chapter. Attempt the assignments. The purpose of the assignments is 10 give you a practice in solving various levels and varieties of problems. Each problem has some important concept which it highlights. When you do a problem from an assignment, make sure that you have completed the study material, have committed the formulae to your memory and have solved the solved problems (most of them on your own before seeing the solution) Do not open the study material to refer to formulae theoretical concepts while doing the assignment problems unless it is absolutely essential to do so. Do full justice to the exercises and assignment problems. Even if you do not get the answer to a problem, keep trying on your own and only approach your friends or teachers afier making lot of attempts. Do not look at the answer and try 10 work backwards. This would defeat the purpose of doing the problem. Remember the purpose of doing an assignment problem is not simply to get the answer (it is only evidence that you solved it correctly) but to develop your ability 0 think. Try 10 introduce twists and turns in given problem to create similar problems. This book is based on our experience over the past few years. This material covers extensively the fundamental principles and concepts involved, solved problems which highlight the application of these concepts, exercises and assignments for practice by the studems. In order to get maximum benefit from this material, ‘word of Advice’ given overleaf has to be carefully followed. The book besides IITJEE will also prove useful to students for other Engineering examinations as well as their school curriculum Wishing you all success. INTEGRATED SCHOOL PROGRAM CHEMICAL KINETICS CONTENTS SYLLABUS > Introduction 1 > Rate of Reaction 2 CBSE: Rates of chemical reactions, Theories of Law of Mass action 5 chemical reaction & mechanism of reaction. 7 Moleculariy 6 > Order of Reaction 7 > Reactions of Various Order 10 > Factors affecting the Rate of a chemical Reaction 17 JEE: Rates of chemical reactions; Order of reactions > Collision theory of Reaction Rate constant; First order reactions; ‘Temperature Rates a dependence of rate constant (Arrhenius equation). YF Radioactivity 4 + Answers to Exercises 36 1 Miscellaneous Ex 38 + Solved Problems 42 Chapter Practice Problems $9 > Assignment Problem o > Answers to CPP 79 > Answers to Assignments Problems 80 INTRODUCTION ‘Chemical Kinetics is the branch of science that deals with rate of reaction, factors affecting the rate of reaction and reaction - mechanism. Different reactions occur at different rate. In fact a chemical reaction involves redistribution of bonds — breaking of bond(s) in the reactant molecule(s) and making of bonds in the product molecule(s). The rate of a chemical reaction actually depends upon the strength of the bond(s) ‘and number of bonds to be broken during the reaction. It takes longer time for the reactant molecules to acquire higher amount of energy which they do by collision. Hence reactions involving strong bond - breaking occur at relatively slower rate while those involving weak bond - breaking occur at relatively faster rate. On the basis of rate, reactions are classified as. Instantaneous or extremely fast reactions ie, reactions with halflife of the order of fraction of second. = Extremely slow reactions ie. reactions with halflife of the order of years. = Reactions of moderate or measurable rate. lonic reactions are instantaneous. If a drop of silver nitrate solution is added to a solution of the chloride salt of any metal or solution of HCI, a white precipitate of silver chloride appears within twinkling of eye. This is because of the fact that in aqueous solution an ionic compound exists as its constituent ions. No bond needs to be broken during the reaction. Hence reaction takes no time to complete, The half life period of an ionic reaction is of the order of 10°" s. Na’ + CI + Ag’ + NO, —> AgCIl + Na’ + NO; Free radicals being very unstable (reactive) due to the presence of unpaired electron, reactions involving free radicals also occur instantaneously. Thus, the reactions, are instantaneous. “CH + Cl, —> CHCI + *Cl “CHs + *CH; —> HC - CH Some molecular reactions involving reactant(s) containing odd electron completes within a fraction of second. The speed of such reactions is attributable to the tendency of the odd electron molecule (paramagnetic in nature) to transform into stable spin-paired molecule (diamagnetic) by dimerisation. An example of such reaction is the dimerisation of nitrogen dioxide into nitrogen tetraoxide as mentioned below. *NO2 + *NO2 —+ N20. There are some molecular reactions which are known to be extremely slow. Their half-lives are of the order of several years. Some examples of the type of reactions are as given below: 4Fe + xH,0+30, —> 2Fe,03.xH;0 [Cr(H:0)_]* + > —> (Cr(H;0).l|*" + H20 Note that the first reaction given above is called “rusting of iron" The second one is not ionic reaction as it appears at the first sight. Here in this reaction it is the co-ordinate bond between "Wie Lid, FITUBE House, 294, Kalu Sarai, Sarvapriva Vihar, New DelN 110016, Ph 46106000, Fax 26513942 ‘website: wow fllfe.com ISPBR & Du2021.CxnCHCK: Chemical kinetics central metal ion i.e. Cr” (acceptor) and water molecule (donor) that is broken and covalent bond between Cr and I that is formed. The halflife of this reaction is in years. Most molecular reactions especially organic reactions occur at measurable rate. The halt-lfe of ‘Such reactions are of the order of minutes, hours, days. Examples of such reactions are numerous. Some of these are given below. CHsCOOC:Hs + H:0 —+ CH;COOH + C3HsOH CyHnO,, + HO —*> C.H,,0,+C,H,,0, ete) (uc) ute) H,02 (aq) —>H,0 + 40, t 2N:0; —+4NO; +0, t NH.NO; (aq) —+2H,0 + N, T In Chemical Kinetics we deal with the rates of only those reactions which occur with measurable rate i.e. which are neither too fast nor too slow. The rates of fast reactions are also determined using lasers. RATE OF REACTION The rate of a reaction means the speed with which the reaction takes place. This is expressed either in terms of decrease in the concentration of a reactant per unit time or increase in the concentration of a product per unit time. Rate of reaction = D&0t€a8e in the concentration of a reactant Time interval or increase in the concentration of a product _ ax . Time interval at The term Ax Means Xi ~ Xingu and At is the amount of time elapsed. For example, a car driver starts his journey at 9.00 AM with odometer reading x miles. At 11.00 AM, he reaches his destination. The odometer reading at destination is y miles. The rate of his travel can be calculated as (distance) distance, ~distance,., Rate = A(time) time,,, ~ time, =X 2 ites nt 11.0-90 2 The above example indicates that the car has been driven with uniform rate but actually it has been driven sometimes faster and sometimes slower depending upon the condition of road. Thus, the overall rate is an average rate and the rate at which the car was moving at any instant, called instantaneous rate. The rate measured over a long time interval is called average rate and the rate measured for an infinitesimally small time interval is called instantaneous rate {In general, for any reaction of the type A+B—>C+D AAI __ 16] _, A{C] _ , s{0) at at att Where [A] signifies the molar conc. of (A) and A[A] stands for the change in molar concentration of A. The negative sign placed before a reaction rate symbol signifies a decrease in concentration of the reactant with increase of time and a positive sign before the rate symbol signifies that the concentration of product increases with increase of time. Average rate of reaction Vtie Led, FITUBE House, 294, Kalu Saral, Sareapriya Vihar, New Delhi 110016, Ph #6106000, Pax 2GSTOO4T ‘website: ww fsjee.com Chemical kinetics isp.pp a pi.2021-C.XICHCK-3 Average rate of reaction The rate measured over a long time interval is called average rate. The rate of reaction (average rate) is defined as the change of concentration of any one of the reactants (or products) per unit time. Change of concentration of one of reactants or products Time taken for the change Average rate of reaction tage Consider the reaction between CO and NO, O+NO, —+CO, +NO This equation shows that one mole of CO reacts with one mole of NO,, one mole each of CO, and NO are formed. The average rate of reaction can be expressed either by decrease in conc, of reactant (CO, NO,) or by the increase in cone. of any one of products (CO, or NO), A(CO) __A(NO,) __A(CO,) _ A(NO) at at at at For the reaction, 2H,O, —»>2H,0+0, When 2 moles of H,O, decomposed, one mole of ©, and 2 moles of HO is formed. The rate of increase in the conc. of O,, therefore is half that of the disappearance of the cone, of HO, and increase in conc. of H,O is the same of the disappearance of the conc. of H,, in the same time interval. 0 210s} _-1.1H.0.1 _ 1 41H,0] a 2 at 2 at In general, for a reaction, nA +n,8—>m,C-+m,D The rate is expressed as: 1 afA _ -1 AB) n, at on, at Thus, Instantaneous rate ‘With the progress of reaction the conc. of reactants decreases while that of product increases. According to law of mass action the rate of reaction decreases moment to moment as shown by graph of rate vs. time. Rate varies from moment to moment so rate of reaction has to be § specified at a given instant of time called instantaneous rate ac fee OF = ESE at Time Where dC is the infinitesimal change in conc. during infinitesimal time interval dt after time t Le. between t and t+ at. Consider a reaction A+B ,To know the rate of reaction at any time t, a tangent is drawn to curve at the point corresponding to that time and it is extended on either side so as to cut the axes, say at the point A and 8. Then Change in the cone. Time Cone. (mole/it) Rate of reactiot a. a = slope of the tangent Time (min.) Thus the rate of reaction at time 10 minutes “Waite Lid, FUTUBE House, 2-4, Kalu Sarat, Sarvapriya Vihar, New DeiRt 1 10016, Ph #6106000, Fax 20513942 — ‘website: wun fltjoe.com Chemical kinetics ISP-BR & DH.2021-C-XIL-CH-CK-4 ————____—O"' C3" NIN@tNeS At = 20x60 = 1200 sec. Ax =0.04 moles 0.04 }200 3.3510 mol lit" sec"* Units of the rate of reaction ‘Since concentration is usually expressed in moles /litre, time is taken in seconds or minutes, the unit of the rate of reaction is moles lit’ sec"'(ML" sec”) or moles lit” min’! (ML min’) or moles lit" hour’* (ML" fr') or moles lit"* day* (ML"d"') or moles lit ' year ' (ML y"). Mlustration 1. Solution: Mustration 2. Solution: Mustration 3. Solution: Mustration 4. Solution: Mustration 5. Solution: Why we prefer instantaneous rate of reaction over average rate of reaction? The rate of reaction decreases continuously with time except for a zero order reaction. Therefore, average rate of reaction has no significance for the reaction, But instantaneous rate of reaction for a given instant of time does not change with time, Define rate of a reaction. Rate of a reaction may be defined as the change in concentration of any one of the reactants or products per unit time. Define specific rate constant. It is defined as the rate of a chemical reaction when the concentration of each reactant appearing in the rate equation is taken as unity. The following reaction was carried out in water. Cl, +2! ——»2Cl +1, The initial concentration of was 0.50mol -' and concentration after 10 minutes was 0.46 mole lit’ Calculate the rate of disappearance of I" and rate of ‘appearance of iodine, (I ] = 0.46 -0.50 = -0.04 mol At = 10.0 -0.0 = 10 min a(t] _ -(-0.04) Rate of dis: fre isappearance of aa aD = 0,004 mol" min" Rate of appearance of iodine = JiRate of disappearance of ) ro (004 = 0,002 mol ¢“'min"* The rate of formation of nitric oxide (INO) in the following reaction is 3.610" mol f's" ANH, (9) + 50; (g) —> 4NO(g) + 6H,0(9) Find the rate of disappearance of oxygen. The rate of above reaction in terms of oxygen and NO are ‘iti Led, FITUEE House, 29-4, Kalu Sarat, Sarvapriya Vihar, New Delhi 110016, Ph 46106000, Fax 26819902 ‘website: wu fljee.com enemicsiikinstice ISP-BR & DH-2021-C-XILCH-CK.5 For the hypothetical reaction: A, + 2B, —>2AB,, write the rate equation in term of the disappearance of B; and formation of AB2. Exercise 2. In the reaction: 58r’,,,, + BO; ,.,, Express the rate of disappearance of Br in terms of formation of Br:. + 6H" ,,, —9Br,,,, +9H,0,, Exercise 3. The reaction, 2N,0;(9)——4NO,(g)+0,(g), takes place in a closed flask. It is found that concentration of NO, increases by 20x10'molf' in 5 seconds. Calculate the rate of the reaction and the rate of change of concentration of Nz0s. LAW OF MASS ACTION “Ata given temperature, the rate of a reaction at a particular instant is proportional to the product of the active masses of the reactants at that instant raised to powers which are numerically equal to the numbers of their respective molecules in the stoichiometric equation describing the reaction’ Active mass. jolar concentration of the substance ‘number of gm moles of the substance Volume in litre “VV where W = mass of substance, V= volume in litres Consider a simple reaction A+B If Cxis the molar concentration or active mass of A at a particular instant, then % 2c, or *-Ke, at Where K is a proportionality constant or rate constant. WFCq=1 then ax Rate = > at = molecular mass in grams. Let us consider a general reaction aA +bB—+Product " PHTUBE House, 29-4, Kalu Sarat, Sarvepriya Vihar, New Delhi 110016, Ph 46106000, Fax 26513992 ‘webatte: waw fltjee.com Chemical kinetics ISP-BR & DH-2021-C-XI-CH-CK-6, ———____"__ NU Rate -(2) ay er (ay 8 Rate of reaction at unit concentration of reactant is called rate constant. The value of rate constant depends on (Nature of reactant (i) Temperature (it) Catalyst MOLECULARITY ‘A chemical reaction that take place in one and only one step i.e., all that occurs in a single step is called elementary reaction while a chemical reaction occurring in the sequence of two or more steps is called complicated reaction, The sequence of steps through which a complicated reaction takes place is called reaction mechanism. Each step in a mechanism is an elementary step reaction The molecularity of an elementary reaction is defined as the minimum number of molecules, atoms or ions of the reactants required for the reaction to occur and is equal to the sum of stoichiometric coefficient of the reactants in the chemical equation of the reaction. Thus, the molecularity of some elementary reactions are as mentioned below Elementary reactions Molecularity PCI, —=PCI, +Cl, 1 H, +1, —=22HI 2 Reaction with molecularity equal to one, two, three etc; are called unimolecular, biomolecular, trimolecular etc. respectively. ‘A complicated reaction has no molecularity of its own but molecularity of each of the steps (elementary reactions) involved in the mechanism. For example; consider the reaction; 2NO+2H, —>»N, +2H,O;which is complicated reaction and takes place in the sequence of following three steps: () — NO+NO==N,0, (fast and reversible) (i) —_N,O, +H, # 5N,0+H,0 (slow) (ii) N,O+H, —N, +H,0 (fast) The molecularity of each step in the mechanism is two, so that we say that the reaction takes in the sequence of three steps each of which is bimolecular. There is another way also. According to which molecularity of a complicated reaction is taken to be equal to the molecularity of the slowest step i.e. rate determining step (f.d.s) in the mechanism. For example, the reaction ROH +HCI—=2 RCI+H,O is said to be unimolecular nucteophil substitution (S,.1). Since the reaction occurs in the sequence of the following three steps and the slowest step i.e. r.d.s. is unimolecular. © RGH+HCI—>ROH, +r (fast) —Sooeee———— ‘Wee Led, PITUBE House, 29-4, Kalu Sarat, Sareapriya Vihar, New Delht 110016, Ph 46106000, Fax 26519940 ‘website: wow ftjee.com ——Shemieal kinetics spar a pin2021.0xNCHCK-7 (ROH, —R* +H,0 (slow) (ii) Re +Cr —>Rel (fast) Reactions of higher molecularity (molecularity > 3) are rare. This is because a reaction takes place by collision between reactant molecules and as number of a reactant molecules i.e. molecularity increases the chance of their coming together and colliding simultaneously decreases. ORDER OF REACTION ‘The mathematical expression showing the dependence of rate on the concentration of reactant is known as rate law or rate expression of the reaction and sum of the indices (powers) of the concentration terms appearing in the rate law as observed experimentally is called order of reaction, To understand what is order of reaction, consider the reaction 2NO(Q) + 2H, (g) —N, (g) + 2H,0(Q) Kinetic experiment carried out at 1100°K upon this reaction has shown following rate data, Experiment Number | [NO](mole dm*) | (H,](mole dm*) | Rate (mole dm® s I 4 5x10 25x10? 3x10" 2 1107 25x10 12x10" 3 413107 5x40" 24x10" From the experiment number 1 and 2, itis evident that rate increases 4 fold when conc. of NO is doubled keeping the conc. of H, constant i.e, Rate x [NO}* when [NO]is constant from experiment number 2 and 3, it is evident that when concentration of H, is doubled keeping the cone, of NO constant, the rate is just doubled ie. Rate = [H,] when [NO]is constant Overall rate x{NOF (H,] Order of reaction with respect to NO is 2 and with respect to H, is 1. The overall order of reaction is 2 + 1 = 3. This order of reaction suggest that the reaction is complicated and it does not occur in single step. In order to explain this reaction following mechanism has been proposed @ — NO+NO===N,0, (fast and reversible) (i) —_N,O, +H, —+N,0+H,0 (slow) (ii) N,O+H, —N, +H,0 (fast) Rate of overall reaction = Rate of step I! = K[N,O, JH] where K = Rate constant of step 1! NO, being intermediate for the overall reaction, its concentration has to be evaluated in terms of the concentration of reactant and this can be done by applying law of mass action upon the equilibrium of step |, Thus, x, - NOl INO oF IN,O,] = K [NOP where K, =equilibrium constant of step |, putting this value of concentration of N,O, in the above rate expression, we get Rate of reaction = K. K,[NOJ* [H,] or Rate of reaction = K’[NOF’[H,] rapriva Vihar, New Delh 110016, Ph 46106000, Fax 26513902 ‘webatte: ww ftjee.com Chemical kinetics ISP-BR & DH-2021-C-XILCHCK.8 Rate of reaction 2-(NO}? (H,] Where K’=K.K, is another constant, rate constant of overall reaction, {n general, if rate law of a reaction represented by the equation mA +nB —»Pr oducts is experimentally found to be as follows Rate x{AT"[B]" Then order w.rt. A =m, order wart. B= A Overall order=m +n !t may be noted that 'm’ may or may not be equal to a and similarly ‘n’ may or may not be equal to b, m and n are experimental values, which really depends upon reaction mechanism and experimental condition, may not be predicted by just seeing the chemical equation of the reaction ‘An example of this is as follows: () 2N,0, —>4No, +0, Rate x [N,O,] Order of reaction is 1 (i) 2NO, —+N,0, +0, Rate x (NO, Order of reaction is 2. Mlustration 6. For a reaction, 2NO, + F, —>2NO,F The experimental rate law ist =K[NO,|[F,]. Propose the mechanism of the reaction. Solution: The rate law isr = K[NO,|[F,], since rate law is proportional to single power of NO» and single power of F2, it implies that only one molecule of NO; and one molecule of Fare involved in the slow step. Thus the various steps are: Step 1 NO, +F, —>NO,F +. (slow) Step 2 NO, +F —>NO,F, (fast) 2NO, +F, —>2NO,F Mlustration 7. The reaction, NO, +CO——+CO, +NO is experimentally found to have a rate given asr=K[NO,}’ Suggest a mechanism consistent with the data. Solution: Step 1 NO, +NO, —" 5NO+NO, Step 2 NO, +CO—* +CO, +NO, Exercise 4. The gas ~ phase decomposition of acetaldehyde. “Pron ono CH,CHO,,, —>CO,,, +CH,,,, follows the rate law Penoe. = KD enjcno) If pressure is in atmosphere and the time is in minutes. Find the units for (i) rate of reaction (ii) rate constar ‘Wii Led, FITUEE House, 29°, Ratu Sarat vreapriga Vihar, New Delht 110016, Ph #6106000, Fax 26519942 ‘website: waw ftjes.com Chemical kinetics ISP-BR & DH-2021-C-XIL-CH-CK-9 Exercise 5. (a) The reaction 2NO,CI —»2NO, + Cl, Proceeds by the following mechanism () NO,Cl "+ NO, +1 (il) NO,CI + C1" NO, + Cl, What is the rate law expression? (2) For the reaction 2NO(g)+0;,(9)—> 2NO, (9) The experimental rate law is r = K [NO] [O,], propose the mechanism of the reaction. Pseudo first order reaction Reaction whose actual order is different from that expected using rate law expression are called pseudo — order reactions, eg RCI +H,0 —+>ROH +HCI Expected rate law: Rate = K RCI] {H,0], Expected order=1+1=2 Actual rate law: Rate = K’ [RCI], Actual order Water is taken in excess; therefore, its concentration may be taken constant. This reaction is therefore, pseudo first order. Similarly, the acid catalysed hydrolysis of ester, viz, RCOOR' + H,O===RCOOH +R'OH follows first order kinetics. Rate = K[RCOOR’] Itis also a pseudo ~ first order reaction. Mlustration 8. If the decomposition of nitrogen (V) oxide 2N,0, —+4NO, +0, Following a first order kinetics () Calculate the rate constant for 0.04 M solution, if the instantaneous rate is 1.410 mole"'s* (ii) Also calculate the rate of reaction when the concentration of N:Os is 1.20 M. (iil) What concentration of NOs would give a rate of2.45x 10° mol ¢'s '? Solution: (i As the given rate is of the first order, therefore, rate = K[N,O.] rate _1.4x10° [N,0.] ~~ 0.04 (ii) Now if the concentration of NOs is 1.2 M, then, rate = K[N,O,] =3.5x10° «1.20 = 4.210% mol e's" (iil) To obtain concentration of NxOs when the rate is 2.4510" mol /"'s Rate 2.45x10° Ie = Seto Imustration 9. The experimental data forthe reaction 2A+B,—+2AB is 5x10 s* =0.7 mol (of 0.7M Experimer “[ieaimoinit [initial rate 1 0.50 1.610 2. 1.00 3.2x10* _ 3. 1.00 3.2x10 "Zod, Kalu Sarat, Sarvapriga Vihar, New Delht 110016, Ph 46106000, Fax 26519942 ‘website: wu fljee.com Chemical kinetics ISP-BR & DH-2021-C-XIL-CH-CK-10————_———_ Solution: Let the rate law equation be, r= K[A]'[B,’ Now, 1, =1.6 «10 =K(0.50)" (0.50) ) =K(0.50)" (1.00)" (iy (1.00)"(1.00)' «i & _K(0.50)'(1.00)' _3.2x10¢ Divide (i) b a wae by ms K(0.50)*(0.50)’ 1.6x10* 1.00)" 100)" 22% 22 y=t (<5) Paray Now divide (ii) by (i) 6 _ K(1.00)'(1.00)" _3.2x10¢ & K(0.50)"(1.00)’ 3.2x10* 1.00)" e (se) <1 221 >x Rate law = ¢=K(Al'[B,]' Exercise 6. Three experiments were carried out for the reaction between Cl; and NO Cl,(g) + 2NO(g)—+ 2NOCI(g), the following data were obtained. int_| initial concentration ialrate 0.01 0.03 0.03 Determine: () the orders with respect to Cl; & NO (il) the rate law. (iif) the rate constant Difference between order and molecularity () Orders an experimental property while molecularity is the theoretical property. (i) Order concerns with kinetics (rate law) while molecularity concems with mechanism. (i) Order may be any number, fraction, integral or even zero, whereas molecularity is always an +ve integer. REACTIONS OF VARIOUS ORDERS @ Zero order reactions A reaction is said to be zero order if its rate is independent of the conc. of the reactants, consider the general reactions: A——>Products Ifitis reac. of zero order Rate =a KIAP =K ord{a] =-K dt Integrating both sides, we get rapriva Vihar, New Delht 110016, Ph 46106000, Fax 26519942 ‘website: ww fljee.com etemicsiiiinetion! ISP-BR & DH-2021-C-XIL-CH-CK-11 (A= -Kt+ 1 @ where | is a constant of integration Att= 0, [A]=IAI, “Al, Substituting this value of lin equation (), we get (Al= -Kt+fAL (i) or Kt =[Al, -fAk K itl -[A} di) ‘Some important characteristics of reaction of zero order. () Any reaction of zero order must obey equation (i). AS it is a equation of straight line (y= mx + c), the plot of [A] versus t will be a straight line with slope = -K and intercept on the conc. axis = Al, as shown in figure Concentiation Rate Time i) Half life period: Half life period (t,,.) is the time in which half of the substance has reacted, (Au tat, substituting those values in 2 Ta equation (ii), we get =Aftay, Ak a= g|IAb-S = Al 2K Unit of K: When [A] Initial concentration [Alp ‘molar conc. _ mol L* Time time = mole Examples of zero order reaction (i) Photochemical reaction between hydrogen and chlorine Hy) +Chig —2—»2HO (ii), Decomposition of N;O on hot platinum surface bie lo N,oO—>n, +40, : 2 (ii) Decomposition of NH in presence of molybdenum or tungsten 2NH, — ML st, + 3H, IMustration 10. Write the integrated rate equation for a zero order reaction ,-BL-(l Solution: rreapriya Vihar, New DelNt T1001" ‘website: wa fljee.com "Fh 46106000, Fax 26519942 Chemical kinetics ISP-BR & DH-202 1-C-KIL-CH-CK-12——————— nn where k = rate constant [Rho = initial concentration of the reactant [R]= concentration of the reactant at time, t. Exercise 7. The following graph is a plot of the rate of reaction versus concentration of the reactant. What is the order of the reaction? ——con Exercise 8. At 250°C, the half life for decomposition of N2Os is 5.7 hr and is independent of initial pressure of N;Os, The specific rate constant is (A) 0.693/5.7 (B) 0.69337 (©) 5.770.693 (0) none First order reaction ‘A reaction is said to be of the frst order i the rate of the reaction depends upon one cone, term only Consider the reaction A—»Products Let ‘a! be the conc. of A atthe start and after time t the cone. becomes (a ~ x), ie., x has been changed info products. The rate of reaction after time "is given by the expression % N, +3H, () [NH,] is very low. (ii) [NHa] is very high. Exercise 10. For a chemical reaction X > Y, the rate of reaction increases by a factor of 1.837, when the concentration of X is increased by 1.8 times. The order of the reaction with respect to X is At (8) 1.5 2 ()-1 Exercise 11. 50% of the amount of a radioactive substance decomposes in § years. The time required for the decomposition of 99.9% of the substance will be (A) 10 years (B) between 10 and 40 years (C) less than 10 years (0) between 49 and 50 years Some important characteristics of first order reaction (@) A change in cone. unit will not change the numerical value of K. Let the new unit be n times the first one 2303, na 2303, a So k= 22038 j6g,, 2.30369, t aay tw (b) Graphical method: 2.303 ing, (AL, t al fog,o(Al, ~l09,0fA] = ss Kt log, (Al = ~~ +109,0{A] log ofA] = 539g *!0dwIAL, Comparing it with y = mx + ¢ T Loo (A) Slope m= as intercept c = log..{Al, (©) Half life period: The time taken for any fraction of the reaction to be completed is independent of the initial concentration “Wate Led, PITRE House, 29-4, Ratu Sarai, Sarvapriya Vihar, New DelNt 110016, Ph 46106000, Fax 26513902 ‘website: ww fljee.com eR emicsiiiinetics ISP-BR & DH-2021-C-XI-CH-CK-15 2.303, [Al Ce tal when the half reaction is completed [A] = ah ._—_—___ 2.303, [A], _ 2.303 tha = log ae = SS va lOO a 92 2 0.6932 L_____s_~ bya Concentration of reactant, Half - Life of an” Order Reaction: To find out the t, for a n'” order reaction where n +1 We ; AL gap = AL cna =P AA caf cB a) will 2 T ap"efal=tt, oF (a) kia [ist -[SE] Jaws = AE f-(2) "Jet 1 ore (2-1) Taal = aa =Kt,, (order net) Therefore for a nth order reaction, the half life is inversely related to the initial concentration raised to the power of (n—1), Note: It can be noted that for a zero order reaction ts: [AL 2k Mustration 13. Define half-life of a reaction. Solution: The half-life of a reaction is defined as the time required for the reaction to be 50% complete. It is also defined as the time during which the concentration of reactant is reduced to one half ofits initial concentration. Exercise 12. The following graph is a plot of tyz and concentration. Whaat is the order of the reaction? Examples of first order reaction (Decomposition of H:0: in aqueous solution 1 —H,0+40, H,0, —H,0+ 5 "Hee Led, FITUBE. iu Saral, Sarvapriva Vihar, New Delhi 110016, Ph 46106000, Fax 26513942 Chemical kinetics ISP-BR & DH-2021-C-XI-CHCK-16- 0) 2HNO, —+2NO +H,0 +20, 40,20, 40, Unit of rate constant Unit ofrate constant * | Garor concenvaton [ A T 1 \ e xtime mole/it =f times mote | where n = order of reaction mtn 14. A first order reaction has a specific reaction rate of 10~ s"’ How much time will it take for 10 g of the reactant to reduce to 2.5 g? | ste" A Solution: Concentration after ‘n’ half life [A] = Al [Ah _ 10 yp !Al 10 42> one 2 fal 25 orn=2 tha = 2593 - 693 10" time required to reduce to 2.5g = 693 x 2 = 1386 sec Exercise 13. — A first order reaction is 78% completed in 60 minutes. Find tes of the reaction. SOME COMPLEX FIRST ORDER REACTIONS. Parallel Reactions: a In such reactions (mostly organic) a single reactant gives ay, two products 8 and C with different rate constants. If we oe assume that both of them are first order, we get -d[A 1 eg fal eky[Al=(&, +h )LA] a @ (3) Let us assume that in a produced e interval, dt, x moles / it of B was produced and y moles /lit of C was ‘“funuee Led, FITUBE House, 29:4, Kalu Sarai, Sarvapriga Vihar, New DelNt 110016, Ph 46106000, Fax 26513992 ‘website: wuw flgjee.com eiemiesiiiinetics ISP-BR & DH-2021-C XI-CH-CK.17 dB] x afc] _y at at at at B to that of C from the start (assuming no B and C in the beginning) is a constant equal to ky/ks Exercise 14. A substrate undergoes a first order decomposition. The decomposition follows two parallel first order reactions as K,=2«10* sec" we 1=2x10" sec —* © K,=8x104 sec K, The % distribution of C and B are (A) 80% & 20% (B) 25% & 75% (C) 20% & 80% (D) 75% & 25% ‘Sequential Reactions: A—*»B—+5C. In this A decomposes to B which in turn decomposes to C. aA. AAAL (1) at oe) k; (A) -kAB] (2) ate) = ke (8) 3) (4) ‘Substituting equation (4) in equation (3) FACTORS AFFECTING THE RATE OF A CHEMICAL REACTION ‘The various factors which influences the rate of reaction are () concentration of reactants (il) temperature (il) nature of reacting substances (iv) presence of catalyst (v) exposure to radiations 1. Concentration of the reactants Greater is the concentration of reactants, more will be the chances of collisions between the reacting particles, consequently, larger is the rate of the reaction. For gaseous reactants as the concentration is related to the pressure, therefore, greater is the pressure more will be number of "Ph 36106000, Fax 36519042 apr ‘website: wuw fljee.com Chemical kinetics ISP-BR & DH-2021-C-XIL-CH-CK-18- ‘molecules per unit volume and consequently, greater will be the rate of encounters between the molecules. 2. Effect of temperature on reaction rate The rate of almost all reactions increase with the increase in temperature, In most of the cases the rate of the reaction becomes almost double for every 10° rise of temperature. This is also expressed in the terms of Temperature co-efficient. Which is the ratio of rate constant of the reaction at two temperatures differing by 10° The two temperature generally selected are 298K and 308 K. Thus (constant at 308 K Temperature coefficient = —— — S20 i Rateconstant at 298K Explanation According to colision theory of reaction rates. Rate of a reaction = f x z Where f = no. of effective collision 2 = frequency of collision Thus, the increase in rate is due to either of the above factors is f orz or due to both of these It may be observed that the increase in the total number of collision per unit volume per unit time (Collision frequency) is not so much responsible for the higher reaction rate as is the inorease in the fraction of effective collisions. Let us, for eg, calculate the increase in collision frequency when temperature increases from 298 to 308 K. As we know that collision frequency is directly proportional to the square root of absolute temperature, therefore, the rate of collision frequencies at these temperatures follows as Zoos _ [308 Zany V288 From the above ratio, it is clear that there is a insignificant increase in the collision frequency. Hence it can not explain the observed increase in the ratio of the reaction with increase in temperature. Lt us now consider the effect of increase in temperature on the number of effective collisions 016 te Te (T4109) T>T Energy corresponding to by activation energy Fraction of molecules: Fraction of molecules. Energy distribution at different temperature Now, as we know that the rise in temperature increases the kinetic energy of the molecules, (:KEx T)therefore the energy distribution curve gets flattened and shifts towards higher energy region. A close revels examination of the curves in the graph clearly reveals that the fraction of molecules possessing higher kinetic energy i.e. energy greater than threshold energy, as indicated by shaded portion becomes almost double and therefore, the rate of reaction almost ‘tice Led, FITUEE House, 29°, Kalu Sarat, Sarvapriga Vihar, New Delhi 110016, Ph 46106000, Fax 26519942 ‘website: wun fltjee.com Chemical kinetics en ae neacatosarerromeel doubles for 10° rise in temperature. Thus, increase in the rate of reaction with increase in temperature is mainly due to increase in no. of collisions which are energetically effective. Not The reaction rate dependence on temperature can also explained by Vant Hoff's pete 2.308R|7, T, Nature of reacting substance The nature of reacting substances affect, the rates significantly. For e.g. the oxidation of ferrous (Fe) by KMnO, in acidic medium is practically instaneous. On the other hand, oxidation of oxalate ions (C,0;2) by KMnO, in acidic medium is comparatively much slower MnO," + 8H” + 5Fe” —+Mn’ +4Fe” + 4H,0 (fast) 2MnO; + 5,07 + 16H" —+2Mn"? +1060, +8H,O (stow) Catalyst A catalyst is a substance, which increases the rate of a reaction without itself being consumed at the end of the reaction, and the phenomenon is called catalysis. There are some catalysts which decrease the rate of reaction and such catalysts are called negative catalyst. Obviously, the catalyst accelerating the rate will be positive catalyst. However, the term positive is seldom used and catalyst itself implies positive catalyst. Catalyst are generally foreign substances but sometimes one of the product formed may act as a catalyst and such catalyst is called “auto catalyst” and the phenomenon is called auto catalysis. ‘Thermal decomposition of KCIOs is found to be accelerated by the presence of MnO>. Here MnO» (foreign substance) acts as a catalyst, 2KCIO3 + [MnOz] —+ 2KCI + 3021 + [MnO] MnO; can be received in the same composition and mass at the end of the reaction. In the permanganate titration of oxalic acid in the presence of H;SO, (acid medium), itis found that the titration in the beginning there is slow discharge of the colour of permanganate solution but after sometime the discharge of the colour become faster. This is due to the formation of MnSO, during the reaction, which acts as a catalyst for the same reaction. Thus, MnSO, is an “auto catalyst” for this reaction. This is an example of auto catalyst. 2KMnO, + 3H;SO, + SH;C;O, —+ K;SO, + BHz0 + 10CO2 + 2MnSO, General characteristics of catalyst = A catalyst does not initiate the reaction. It simply fastens it. = Only a small amount of catalyst can catalyse the reaction. = A Catalyst does not alter the position of equilibrium i.e. magnitude of equilibrium constant and hence AG® It simply lowers the time needed to attain equilibrium. This means if a reversible reaction in absence of catalyst completes to go to the extent of 75% till attainment of equilibrium, and this state of equilibrium is attained in 20 minutes ‘Hie Led., FATUBE House, 29:4, Ratu Sarai, Sarvapriya Vihar, New DelN 110016, Ph #6106000, Fax 20513902 ‘webatte: wow ftjee.com Chemical kinetics ISP-BR & DH-2021-C-XI-CH-CK-20- then in presence of a catalyst also the reaction will go to 75% of completion before the attainment of equilibrium but the time needed for this will be less than 20 minutes. = A catalyst drives the reaction through a different route for which energy barrier is of shortest height and hence E, is of lower magnitude. That is, the function of the catalyst is to lower down the activation energy. E, = Energy of activation in absence of catalyst. Ea = Energy of activation in presence of catalyst. Ea ~ E', = lowering of activation energy by catalyst. If k and kea be the rate constant of a reaction at a given temperature T, and E, and E’, are the activation energies of the reaction in absence and presence of catalyst, respectively, the ka ACE k Aen T Kea = git unr Ket = glts-ts k Since E, > E,' So Kea > k. the ratio “2 gives the number of times the rate of reaction will k increase by the use of catalyst at a given temperature and this depends upon E, -E, Greater the value of Es -E,, more number of times ka is greater than k. ‘The rate of reaction in the presence of catalyst at any temperature T, may be made equal to the rate of reaction in absence of catalyst but for this sake we will have to raise the temperature. Let this temperature be T> then eth GSR orfh = = Mlustration 18. A hydrogenation reaction is carried out at 500K. If the same reaction is carried ‘out in the presence of catalyst at the same rate, the temperature required is 400K. Calculate the activation energy of the reaction if the catalyst lowers the activation barrier by 20 kJ/mole. Solution: By Arhenius equation, K= Ae Let E, of the reaction in absence of catalyst = xKJ/mole E, for the reaction in presence of catalyst = (x - 20) Ku/mole K =e 5 0 (e-1) K=Ae i) By equation of () and (i) x _ (K-20) > 500 ~ 400 100 kJimole ‘“jatiee Led, FITUBE House, 25-4, Kalu Sarat, Sarvapriga Vihar, New DelN 110016, Ph 46106000, Fax 76513082 ‘webatte: ww ftjee.com etemicalkinetics ISP-BR & DH-2021.C-XI-CHCK-21 Exercise 15. () Which reaction will have the greater temperature dependence for the rate constant, one with small value of E, or one with large value of E,? (i) The rate of a particular reaction doubles when temperature changes from 27°C to 37°C. Calculate energy of activation of such a reaction. COLLISION THEORY OF REACTION RATES: According to collision theory, a reaction takes place because the molecules collide with each other. The number of collision that takes place per second per unit volume of the reaction mixture is called collision frequency. At ordinary temperature and pressure the value of collision frequency is so high (= 10* to 10" in a gaseous reaction) that if all the collisions were effective, the reaction should be completed in fraction of seconds. However, in actual practice, that is not 0. This is explained on the basis of two factors. (i) Energy factor For a collisions to be effective, the colliding molecules must have energy more than @ particular value, The minimum energy which the colliding molecules must have in order that the collision between them may be effective is called threshold energy. Thus at ordinary temperature and pressure, most of the molecules may not posses energy equal to or greater than threshold value. A collision between high energy molecules overcomes the repulsion and bring the formation of an unstable molecule cluster, called the activated complex, ‘Activated complex tiated comet Energy Energy Reactants a8 Products cro Progress of reaction Progress of reaction (Exothermic) (Endothermic) Thus, every chemical reaction whether exothermic or endothermic has an energy barrier which has to be overcome before reactants can be transformed into products. If the reactant molecules have sufficient energy, they can reach the peak of the energy barrier after collision and then they cchange into products. If the activation energy for a reaction is low, the fraction of effective collisions will be large and reaction will be fast. On the other hand, if the activation energy is high, then fraction of effective collisions will be small and the reaction will be slow. When temperature is increased, the number of active molecules increases, i.e., the number of effective colisions will increase and the rate of reaction will increase Activation energy E5 = Ecactvated compes) Exground state) AH = activation energy of forward reaction — activation energy of backward reaction. (ii) Orientation factor In some cases it is found that even if a larger number of colliding molecules have energy more than threshold value, still the reaction is slow. This is because of improper orientation of the colliding molecules at the time of collisions. i, Sarvopriya Vihar, New Delhi 110016, Ph 46106000, Fax 26813907 ‘website: war flgjee.com Chemical kinetics ISP-BR & DH-2021-C-XIL-CH-CK-22. SB Be 25— 33— 3... Rate of reaction is directly proportional to the number of effective collisions Rate = = cotision frequency «fraction of effective molecules = z= f According to kinetic theory of gases, the fraction of molecules having energy more than a particular value, E at temperature T is given by f=ee" Rate = ze"®/"" As rate of reaction is directly related to rate constant K, we can also write K= zen Arrhenius Equation The variation of equilibrium constant of a reaction with temperature is described by Van't Hoff equation of thermodynamics which is as follows: dink, aH qt RT I ky and kp be the rate constants of forward reaction and backward reaction, respectively then K, = kk. Further, AH = Ea; ~ Ea Putting these in the above equation we get, dink, dink, &,, “dt dT) RT RT? Splitting into two parts din E, <*.+Z (For FR) Te +2 (FOrFR) aT dink, _ &, Oar yet ForBR) where Z is constant Arthenius sets Z equal to zero and without specifying FR and BR, he gave the following equation called Arrhenius equation. dink _ Ea “dt RT From this equation it is evident that rate of change of logarithm of rate constant with temperature depends upon the magnitude of energy of activation of the reaction. Higher the Ea smaller the rate of change of logarithm of rate constant with temperature. That is, rate of the reaction with low E, increases slowly with temperature while rate of the reaction with high E, increases rapidly with temperature. It is also evident that rate of increase of logarithm of rate constant will go on decreasing with increase of temperature. Integrating Equation () assuming Ea to be constant we get, @ "Ate Led, FITUBE House, 25-4, Kalu Sarat, Sarvapriya Vihar, New DelN 110016, Ph 46106000, Fax 26513942 ‘website: wu flgjee.com eh emicalEinetice ISP-BR & DH-2021-C-XIL-CH.CK-23 (ii) cc) Equation (ji) integrated form of Arrhenius equation. The constant A called pre-exponential factor is the frequency factor since it is somewhat related with collision frequency. It is a constant for a given reaction. From Equation (i) itis evident that as T -> <0, k > A. Thus, the constant A is the rate constant of reaction at infinity temperature. The rate constant goes on increasing with temperature. So, when T approaches infinity, k will be maximum. That is to say, A is the maximum rate constant of a reaction. Itis also to be noted that the exponential term i.e. e***" measures the fraction of total number of molecules in the activated state or fraction of the total number of effective collisions. If nes and n be the number of molecules of reactant in the activated state and the total number of molecules of the reactant present in the reaction vessel respectively, then %, enn 1 Equation (ji) may also be put as . Ea )1 i wr (-E2)! gn Since > a= and logA both are constants for a given reaction. So from equation (iv) itis evident E._ and intercept equal that a pot of og k vs willbe a straight ne ofthe slope equal to — 2303R to log as shown below. Cy tan(180-8) =- oa OA SA 2.303R op? Thus, from this plot E, and A both can be determined accurately. If ky & kp be the rate constant of a reaction at two different temperature T; and T» respectively then from equation (iv), we may write B41 1 tog ks A+logA and log k + stoga OO ~za0aR TPA ANd NOK: ree Subtracting former from-the latter we get vogkt= [11 eo 34, 2a0aR (7, 7, Wah the help of this equation it is possible to calculate E, of a reaction provided, rate constants of reaction at two different temperatures are known. Altematively one can calculate rate constant of fa reaction at a given temperature provided that rate constant of the reaction at some other temperature and also E, of the reaction is known. Sarvapriga Vihar, New Delhi 110016, Ph 6106000, Fax 26519942 ‘webatte: waw ftjec.com Chemical kinetics The activation energies of two reactions are Ea, and Fa; with Ea, > Ea;. if the temperature of the reaction system is increased from T; to Tz, predict which of the following alternative is correct? Ki ke Ki (A) t= “2 (B) ay Ky ky io ky ki KS ky Kia ke (D) kk Oe lExercise 17. A catalyst lower the activation energy of the forward reaction by 20 kJ mol". It also changes the activation energy of the backward reaction by an amount (A) equal to that of forward reaction (8) equal to twice that of the forward reaction (C) Which is determined only by the average energy of products (0) Which is determined by the average energy of products relative to that of reactants RADIOACTIVITY © Henri Becquerel in 1896 discovered that rays emitted from Uranium compounds had the properties of affecting photographic plates, ionising air and penetrating through substances. The spontaneous emission of such radiations is known as radioactivity and the elements emitting these radiations are called radioactive elements. Rutherford and Soddy (1903) gave an ingenious interpretation of the radioactive processes and the origin of a. and fb emissions on the basis of neutron-proton theory. (i) The atomic nucteii of radioactive elements are unstable. (ii) The nucteii of radioactive elements are undergoing a process of disintegration forming atoms of new elements called daughter elements which are distinct in physical and chemical properties from parent elements. (i) « and 8 particles are ejected from the nucleus. (iv) rays are secondary products of atomic disintegration. Titiee Led, FITUEE House, 294, Kalu Sarat, Sarcapriya Vihar, New Delh 110016, Ph #6106000, Fax 26513942 ‘webotte: wu fljce.com eh sinicelsinetics ISP-BR & DH-2021-C-XILCH-CK-25 Radioactivity is found to be of three types o, f and y radiations. particles ___ B particles pray. tt it i caries two units positWe | & caries unit negatve | TREY are electromagnetic 1. | charge and four units waves with very short wave charge with no mass. mass. length. Itis represented as helium 2. | nucleus.$He or Heions | itis represented as *e | Itis represented as 2y (He"’) They have high ionising | lonisation power is less 5 | power. than that of « particle. ee ecceaey ,.. | Velocity is less than that | Velocity is the same as that of qT « part 4. | The wgoaty of particle's | right. it varies trom 2.96 | Voht to 2.83 x 10° oms* viz.3 x10" cms * g, | They have low penetrating | Penetration power is 100 | Penetration power is 100 times power. times that of « particle _| that of p particles. The range ie. the distance upto which radioactive | Range is more than that 6 Range is more. Properties can be shown | of « particle. is very small, 8 - 12 om. 7 | They cause luminescence | very ite effect on zine | Very litle effect on zine ‘on zinc sulphide screen. __| sulphide screen. sulphide screen When an « partic is When af particle comes emitted, atomic number | out, atomicno increases | There is no change in atomic 8. | decreases by 2 units and | by one unit, mass number | number or mass number when mass number by 4 units. | remains unchanged. a y-ray is emitted IX SEY +} He pX oh Y +e. flected i Deflected to a greater a, | Deflected in magnetic eee Not deflected in magnetic field field extent in magnetic field TYPES OF RADIOACTIVE DECAY : Nuclear change Usual Types of decay | Radiation | Mass ee Number | condition (Alpha emission | sug _ - _ ae (a) ~ (i) Beta emission ce | intone | a 0 | Stoo tage - (B) —_|2 «iy corer erin ce | pain we | 4 0 | Stoo mat Taine Lea, PITUEE House, 294, Rate i, Sarvapriya Vihar, New Delhi 110016, Ph 46106000, Fax 26819902 ‘webatte: war fljee.com Chemical kinetics ISP-BR & DH-2021-C-XILCH-CK-26- ‘peo! N (iv) Electron capture | X-rays | jpHe> in = — | Ft00 smai (¥) Gamma emission Excited Y — O 0 @ : nucleus #8U—oa' Th+$ He emission) BATH Pa +°, (8) (®- emission) Tht 2 They (y- emission) Ilustration 16. Define radioactivity. Who discovered natural radioactivity? Solution: _Radioacitvity is a phenomenon of spontaneous emission of invisible radiations by certain substances. Natural radioactivity was discovered by Henri Becquerel. Exercise 18. ~ : How does the temperature changes the radioactivity of an element? CAUSE OF RADIOACTIVITY Except in the case of ordinary hydrogen all other nuclide contain both neutrons and protons. A look at the stable nuclides show that the ratio N/P (neutron/proton) in them is either equal to 1 or more than 1. The ratio is = 1 in all the light stable nuclides up to calcium (:3Ca) and thereafter the ratio is ‘greater than 1 for heavy nuctides. bs 100 The stable nuclides lie within the shaded area which is called the region or zone of stability. All those nuclides falling outside this zone are invariably radioactive and unstable in nature. Nuclides falling above the stability zone have an excess of neutrons while those lying below have more protons. Both of these cause instability. These nuclides attain stability by making adjustment in the NIP ratio. Mlustration 17. How can we increase the NZ ratio is ;He? The N/Z ratio can be increased by (Emission of f° Solution: 3He—i He (i) K- capture 3He +°,e—>; H+ X ray ‘THEORY OF RADIOACTIVE DISINTEGRATION Rutherford and Soddy, in 1903, postulated that radioactivity is a nuclear phenomenon and all the radioactive changes are taking place in the nucleus of the atom. They presented an interpretation of the radioactive process and the origin of radiations in the form of a theory known as theory of radioactive disintegration. ‘The disintegration process may proceed in one of the following two ways: (a) a-particle emission: When an a-particle [He] is emitted from the nucleus of an atom of the parent element, the nucleus of the new element, called daughter element, possess “yet Led, FITUEE House, 29:4, Ratu Sarai, Sarvapriga Vihar, New DeiNt 10016, Ph 46106000, Fax 26513902 ‘website: wun fljee.com Chemical kinetics ISP-BR & DH-2021-C-XI-CH.CK-27 atomic mass or atomic mass number less by four units and nuclear charge or atomic number less by 2 units, because a-particle has mass of 4 units and nuclear charge of two units Parent element —=* + Daughter element Atomicmass — W w-4 Atomic number Z Z-2 ea iRa —> Roe {He Bans tt BU > B'thatHe EPo —> E'Pb+{He (b) #particle emission: {i particle is merely an electron which has negligible mass. Whenever a -particle is emitted from the nucleus of a radioactive atom, the nucleus of the new element formed, possess the same atomic mass but nuclear charge is increased by one unit over the Parent element. f - particle emission is due to the result of decay of neutron into proton and electron sn ip +s The electron produced escapes as a p-particles leaving proton in the nucleus. Parent element —S_ Daughter element Atomicmass = W w Atomic number Z z+ 9. i'Pb—> 2Bi +e Th BPaste e Mlustration 18. Arrange the alpha (a), beta (f) and gama (x) radiations in the increasing order of the properties indicated: (i) penetrating power (il) ionizing power (ii) extent of deflection in @ magnetic fea, Solution: —_()_ Increasing penetrating power order is a < B < y. (ii) Increasing ionizing power order is y < B < a. (iil) Increasing extent of deflection in a magnetic field is y g2P bx (He!) + ye") Equating mass number in both sides 238 = 206 + 4x+y Solution: Equating atomic number on both sides: 92 = 82 + 2x y (-1) = 82428 y(t) ny 56 Mustration 20. The atomic mass of thorium is 232.18 and its atomic number is 90. In terms of its radioactive disintegration six a and four f - particles are emitted. What is the ‘mass number and atomic number of product? Solution: goTh?? —__, ,2" + 6a + 48 Equating mass number in both sides 232= m+ 6x4 + 4x0 (Mass no of Th = 232) m =208 Equating, at no, in both sides: 90=A+2x6+4x(1) A=82 Zhas atom no = 82 ‘Mass no = 208 RATE OF DISINTEGRATION AND HALF-LIFE PERIOD The radioactive decay of the different radioactive substances differ widely. The rate of disintegration of a given substance depends upon the nature of disintegrating substance and its total amount. The law of radioactive disintegration may be defined as the quantity of radioactive substance which disappears in unit time is directly proportional to the amount of radioactive substances present or yet not decayed. Rutherford introduced a constant known as half - life period. It is defined as “time during which half the amount of a given sample of the radioactive substance disintegrates” Half life periods vary from billions of years for some radio isotopes to a fraction of a second. Half life period is represented as tya. Let the initial amount of a radioactive substance be No. After ‘one half life period (ty) it becomes = No/2. After two half life periods (2tva) it becomes = No/4 and after n half life periods (nt...) it becomes = (sy No. Thus, for the total disintegration of a radioactive substance, an infinite time will be required. ‘Amount of radioactive substance left after n half life periods N = (iy N,, and total time T =n x tua. Where mis a whole number. Mustration 21. A radioisotope has tye = 5 years. After a given amount of decays for 15 years, what fraction of the original isotope remains? "Hew Deiht 110016, Ph 46106000, Fax 26513962 ‘website: wu flgjee.com eR emicsiiei netics ISP-BR & DH-2021-C-XIL-CH-CK.29 Solution: tia = 8 year t= 15year_ a i Let original amount be No and amount after time t be N. (ty No(y (ty 4 w=(3) Ny (3) -(3) 3 Hence (%)" of the original amount left. Mustration 22, The half life period of ,,P;'° is 140 days. In how many days 1 gm of this isotope is reduced to 0.25 gm? Solution: Original quantity (No) = 1 gm Final quantity of isotope i-) n= matya = 2x140= 280 days integration constant: A chemical reaction whose rate varies directly to the concentration of ‘one molecular species only, is termed as first order reaction. Radioactive disintegration is similar to such a chemical reaction as one radioactive species changes into other. This can be represented as AB ‘Suppose the number of atoms of a radioactive substance present at the start of observation, i. when (= 0, is No and after time t the number of atoms remaining unchanged is N. At this instant of very small number of atoms dN disintegrate in a small time at; the rate of change of A into B is, given -* ‘The negative sign indicates the number of atoms decreases as time increases. Since rate of disintegration or change is proportional to the number of atoms present at that time, the relation becomes. (i) "1s called the disintegration constant or decay constant. Evidently, -SY = 7. wo a= 4 second, 2.2 ai) Thus, %. may be defined as the fraction of the total number of atoms which disintegrates per second at any time. Integrating equation @, f= 7fat or, log N= t+ Ww) where C is the integration constant. Tatiea Lea, FUTJEE House, 294, Chemical kinetics ISP-BR & DH-2021-C-XI-CHCK-30- When t = 0, N= No Putting values in equation (iv) = log No= Putting the value of C in equation (iv) = log N = 7t—log No or, fog No log N= At N, 2.303 logo or, 09. 2.303, ON, 7, = 2808 ogNo or, 8 “ This equation is called kinetic equation and is obeyed by first order reactions. Relationship between half life period and radioactive disintegration constant Whent Ne 2 Putting the values in equation (v) 2303 4g, Ne = 23039. 9 tes NZ ty 0.693 So, [+ logie2 = 0.3010} 0.693 IF tes Thus, half life period of a given radioactive substance does not depend on the initial amount of a radioactive substance but depends only on the disintegration constant of the radioactive element Mlustration 23. What mass of C — 14 isotope will have an activity equal to one curie? Given that the half lite period of C - 14 is 5730 years. Solution: _Rate of disinitegration (-%) = 1 ourie = 3.710" dps 0.693 ; 5730x365 x 24x 3600 . 0.693, , 5730x365 x 24% 3600 or N= 9.6510" atoms ‘Amount of C ~ 14 in grams "4 9.05410" 02244 ome 6.023 x 10’ Mlustration 24, The half lite period of .,1'* is 60 days. What percent of original radioactivity would be present after 180 days? Tice Led, PITUBE House, 29-4, Kalu Sarai, Sarvopriya Vihar, New Delhi 110016, Ph 46106000, Fax 26519942 ‘weboite: wow ftfee.com Chemical kinetics ISPBR & DH2021.CXICHCK31 Solution: ty = 60 days, t= 180 days Applying the formula N = % N14 We get N= Ne. sane 2 8 ny = toon oft, = 125% Exercise 19. (The half - life of tritium ,H° is 12.4 years. Calculate its decay constant in S"'. (i) The half - life period of a radioactive element is 27.96 days. Calculate the time taken by a given sample to reduce to 1/8" of its activity. Average life Period (T) Since total decay period of any element is infinity, it is meaningless to use the term total decay period for radioelement. Thus the term average life is used which is determined by the following relation sum of lives of the nuclides Average life 19 eT = oral number of nuciides Relation between average life and half - life Average life(T) of an element is the inverse of its decay constant, ie., tet ‘Substituting the value of Average life (T) = 1.44 x Half life (ya) Mustration 25. Prove that time required for 99.9% decay of radioactive species is almost ten times of its half life period. 2.303 |. No log Se Solution: No = 100, N= 100 -99.9= 0.1 = 2.303, 100 2.303 toon fog = tosoy = x3 =6.909 G t =, 08 g7 = ay 2) = 0.693 UNITS OF RADIOACTIVITY The standard unit of radioactivity is curie (C) which is defined as that amount of any radioactive material which gives 3.7 x 10°° disintegrations per second (dps). ie., 1 = 3.7 x 10"° dps. Wales Led, FITUEE House, 29 Ph 46106000, Fax 6519002 Chemical kinetics ISP-BR & DH-2021-C-XIL-CH-CK-32- Now a - days, the unit curie is replaced by Rutherford (rd) which is defined as the amount of a radioactive substance which undergoes 10° dps; i.e., 1 rd = 10° dps. 1C=3.7 «10 dps = 37 x 10° rd 1mC=3.7x 10" dps = 37 rd However, in SI system the unit of radioactivity is becquerel (Bq) 1Bq = 1 disintegration per second 10° Bq = 1 rd 3.7 «10° Bq =1C Mlustration 26. Define one curie? Solution: One curie (Ci) is defined as the amount of radioactive isotope that gives 7 x 10° distintegrations per second. Radioactive Equilibrium ‘According to the theory of radioactive disintegration, the product of a radioactive disintegration may also be radioactive and the rate of disintegration of the daughter element depends upon the amount of it present at different time. When steady state is reached, (3). OF, aN = RaNe Where 7.: and 7. are the disintegration constants for the decay of parent and daughter radioactive species and N; and N2 are the number of atoms disintegrating at a given time. thus, Ne (halt life) N ‘(half life) Activity: Activity is the rate at which the radioactive substance decays 0.693, mass ta at mas Specific Activity: Activity of the unit mass of radio isotope. Illustration 27. Sr shows f- activity and its half- life period is 28 years. What is the activity of sample containing 1 gm of Sr? Solution: Activity = 2N = 0.688 ___, 1 6.02310 = 147.89 curie 28305 x 24~60=60 "80 Let the radioactive element ‘A’ decay to B and C in two parallel paths. De —*+8 Activity = ZN = l Parallel Pat ats L446 Decay constant of ‘A’ = Decay constant of B + Decay constant of C ta® Gat 2) (Friction of B) = ——®— Cathe) By heh Ora) he (Frication of C) “ue Led, FITUEE House, 29, Kalu Sarat, Sareapriya Vihar, New Delhi 110016, Ph 46106000, Fax 2681994 ‘website: wnow fljee.com Chemical kinetics ISP-BR & DH-2021-C-XIICH-CK-33 Maximum yield of daughter element Let A——+B (daughter element) If Zs and .g are decay constants of ‘A’ and ‘B’ Maximum activity time of daughter element can be calculated as 2.303 14, Qe - Ac) Number of atoms of daughter element B after time t: (Nal ae Ate!) Nuclear Fission: It is a nuclear reaction in which a heavy nucleus splits into lighter nuclei and energy in released = Californium - 252 decays both by alpha emission (97%) and by spontaneous fission (3%) During spontaneous fission, the nucleus splits into two stable nuclei plus several neutrons. ct —> [Ba + Mo + 4in > When a neutron strikes 2°U nucleus, the nucleus splits into roughly equal parts, giving off ‘several neutrons: InsZFU—oixe + Bsr + Ain or, 8a + kr + ln Les + BRD + 2in = Ifthe neutrons from each nuclear fission are absorbed by other uranium - 235 nuclei, these nuclei split and release even more neutrons. Thus a chain reaction can occur. or, Spallation: Bombardment with high energy charged particles can break some target nuclei into several smaller nuclei with the emission of a large number of nucleus (10 to 20 or more). These are called spallation reaction. Bas +7H—> $Mn +9'H + 12in BPU+ tHe — iW + 20!H+35hn Nuclear fusion: In this type of nuclear reaction, certain light nuclei may fuse together with the liberation of tremendous amount of energy. To achieve this, the colliding nuclei must possess enough kinetic energy to overcome the initial repulsion between the positively charged cores. This energy may be made available by raising the temperature of the reacting system to several million degrees. Such reactions are therefore also known as thermonuclear reactions. 7H +3 H— tHe +} +17.6 Mev The energy of a fusion process is due to mass defect (converted to Binding Energy). The high temperature required to initiate such reactions may be attainted initially through a fission process. Artificial Radioactivity: Bombardment of stable elements with high energy o-particles, protons, neutrons, deutrons or y-rays produce radioactive nuclide. These radionuclide do not occur naturally and may be called man - made or artificial. The radioactivity exhibited by these artificial radio nuclide is referred to as artificial radioactivity or as induced radioactivity. Al + fHe —>» %P + gn RNa + jn —> Mg + Tee Led, FITUEE House, 20-4, Kalu Sarat, Sarvapriya Vihar, New Delhi 110016, Ph 46106000, Fax 26510942 ‘webatte: wow flfee.com Chemic. kin ics ISP-BR & DH-2021-C-XILCH-CK-34- Binding Energy and Packing fraction The nearest integer to the mass of a nuclide is called the mass number of that isotope. The difference between the actual isotopic mass and mass number is termal as mass defect of the nuclide. Thus Am (mass defect) = isotopic mass - mass number ‘Aston introduced a term "Packing fraction" for each nuclide to compare their mass excess, it was defined as: Packing fraction ()= 108 Binding energy (8) = Am x c? Binding energy of the nucleus is equal to the energy required to split the nucleus into its component nucleons. Nuclear binding energy is generally expressed in MeV (mega electron volt), Since 1 a.m.u. = 1.66 x 10” Kg (B) for one a.m.u. = 1.66 x 1077 x (3x10°)°0 = 1.66 x 107” «(3x 10°)*J 1.60x10°*J/eV (®) = aM(namu) «931 Mev B __ total binding energy A total number of nucleons 1 x 10° eV= 931 MeV Binding energy per nucleon (B) Applications of Radioactivity (i) Radio Carbon Dating: The application of the C - 14 dating is based on the fundamental assumption that the intensity of cosmic ray and hence of '“C in the atmosphere has been remaining constant over many thousands of years. This gives the initial activity of C - 14 ‘corresponding to the time when the plant or animal died and assimilation of radio active carbon ceased to continue MN + jn—> $C + 1H Fo > YN + %e 2.30 original activity (N,) Age = final activity (N) (ay orN=No | = \2 where y= Uti (ii) Rock Dating: It is based on the kinetics of radioactive decay. It is assumed that no lead was initially in the sample and the whole of it came from the uranium, Initial radioactivity = (U] = No in terms of gm atoms Final radioactivity = (U} ‘We have assumed that due to high value of tye of uranium its amount remains unchanged N,_(Ul+IPO) _, (Pb) NOU u 2.303, /N,)_ 2.303 (Pb) Thus t 7 ia) tor too[1+ Pal] TWiiee Led, PITUBE House, 25-4, Kalu Sarat, Sarvopriya Vihar, New Delhi 110016, Ph 46106000, Fax 20519942 ‘website: wunw fltjee.com Chemical kinetics ISP-BR & DH-2021.0XI1CHCK.35 or [1-€2] =e ™ Where y zt | hence t is calculated, Mlustration 28. The amount {*C isotope in a piece of wood is found to be one fifth of that present in fresh piece of wood. Caloulate the age of wood (tia = 8577 year. 2.303, No _ | _ 2.903%ty24,NO (\¢ ..9:698 Solution: t= 2203 jaghO y= 2808 stuajgg ho {k olution: KON et O63 ON te ) No |, 2:303%5577 Given N= NO p= 2808%5577igg 5 wenn 8 id _2.303x5577 x = 1295: DeGa «0.6989 = 12953 years Mustration 29. A sample of uranium mineral was hound to contain “Pb and **U in the ratio of 0.008:1. Estimate the age of the mineral (half-life of *U = 4.5110" yeas) Souter: Weknoatt= 222 6 [1-P2 | 2.303xtya{,_ ™Pb] _ 2:903x4.5x10" 1, | 0.08/208 0693 BU 0.693, 17238 = 5.94510" years sreapriya Vihar, New DelNt 110016, Ph 46106000, Pax 26510942 ‘webstte: wow flgee.com ISP-BR & DH-2021.C-XIL-CH-CK-36: Exercise 1: Exercise 2: Exercise 3: Exercise 4: @ (i) Exercise 5: (@) (b) Exercise 6: Exercise 7: Exercise 8: Exercise 9: @ di) Exercise 10: Exercise 11: Exercise 12: Exercise 13: Chemical kinetics ANSWER TO EXERCISES A, + 2B, —>2AB, 2 a Rate of disappearance of Br = Zrate of formation of Br, Rate of reaction = 1x10 mol £"'s" Rate of disappearance of NOs = 2x10” mol 's* Rate of a reaction = atm (min) Rate constant = atm”? min r= K[NO,Cl] Step1 NO +0, NO, (fast) Step2 NO, +NO===NO, +NO, (slow) Rate = K[Cl,][NO[’,K = 1.210? mole“%t? sec? As rate is independent of the concentration the order of the reaction is zero. (A) ‘At very low concentration of NH). ‘At very high concentration of NH). (B) 0) The order of the reaction is one, as ty is independent of the concentration of the reactant, 30 minutes Ted, FATUEE House, 29-4, Ratu Sarat, Sarvapriva Vihar, New Delhi 110016, Ph 46100000, Fax S6S19040 ‘website: waw ftjec.com Exercise 14: Exercise 15: wo i) Exercise 16: Exercise 17: Exercise 18: Exercise 19: @ di) Chemical kinetics (A) ISP-BR & DH-2021-C-XILCH-CK-37 Reaction with larger E, value has greater dependence on temperature, E, =53.6 kJ/mole (8) (A) Radioactivity of an element is independent of temperature. 1.7109 S" 83.9 days ‘webalte: wow flljeecom "Fh 46106000, Fax 26619942 ISP-BR & DH-2021-C-XI-CH-CK-38- Exercise 1: Exercise 2: Exercise 3: Exercise 4: Exercise 5: Exercise 6: Exercise 7: Exercise 8: Exercise 9: Exercise 10: Exercise 11: Exercise 12: Exercise 13: Exercise 14: Exercise 15: Chemical kinetics MISCELLANEOUS EXERCISES The plot of logiR] versus time is a straight line with slope equals to 2.303 x 10° s"’ What is the rate constant of this reaction? The reaction is first order with respect to A, second order with respect to 8 and ‘some order with respect to C. The overall order of the reaction is three half. Write the rate law and find the order of a reaction with respect to C. Define activation energy. What is its units? What is the value of the rate constant, when T approaches infinity in the Arrhenious equation? Define collision frequency factor. Explain what do you understand by transition state, What do you understand by the term ‘activated complex’? Define molecularity. The pressure of a gas decomposing at the surface of a solid catalyst has been measured at different times and the results are given below: Us 0 100 200 300 piPascal 4.010? 3.5x10° 3.010" 2.5%10° Determine the order of reaction, its rate constant and half-life period. The rate of decomposition of H,O2 at a particular temperature is measured by titrating the solution with acidic KMnO, solution, Following results were obtained: Time in minutes 0 nr) Volume of KMnO« in ce 228 138 83 ‘Show that is a first reaction. Calculate k for the reaction. Differentiate between the rate of reaction and the rate constant. Define order of a reaction. Can it be a fractional value? If yes then give an ‘example of a fractional order reaction. Differentiate between order and molecularity of a reaction. How would you compare chemical reactions and the nuclear reaction? Calculate the rate constant of a reaction at 293 K when the energy of activation is 103 KJ mot" and the rate constant at 273 K is 7.87 * 10” sec” Tali Led, FITJEE House, 294, Kalu Sarai, Sarvapriya Vihar, New DelNt 10016, Ph #6106000, Fax 26813942 ‘website: wawfitjee.com Exercise 1: Exercise 2: Exercise 3: Exercise 4: Exercise 5: Exercise 6: Exercise 7: Exercise 8: Exercise 9: Chemical kinetics ISP-BR & DH-2021-C-XILCH.CK.39 ANSWER TO MISCELLANEOUS EXERCISES As the plot of log[R] versus time is a straight line the slope is equal to -k 2.303 k= -2.303 x slope k = 2.303 x (~ 2.303 x 10%) 30 x 107 s* slope The rate law is r= K{A]' [8] [C]’ where x = order w.r.t, C. The given is 1+2+x = 3/2 -3 soimecrrart co? anterior The activation energy is defined as the excess energy (over and above the average energy of the reactants) required by the reactants to undergo chemical reactions. It is expressed in J mor’ As Ton, 290 08" ot and k= Ae atT—> x, k The colision frequency factor is defied as the number of collisions between the reactant molecules A and B per unit volume per unit time divided by the Nx and Ne, the number of molecular per unit volume of A and B respectively. It is denoted by Zu. When colliding molecules possesses the kinetic energy equal to E,, the atomic configuration of species formed at this stage is different from the reactant as well as the products. This stage is called activated state or transition state, Activated complex refers to the specific configuration of a transition state which is supposed to be in equilibrium with the reactant molecules and has all the attributes of a normal molecule except that one of the vibrational degree of freedom along the reaction is converted into transitional degree of freedom. The molecularity of a reaction is defined as the number of reacting molecules which colloide simultaneously to bring about a chemical reaction. The rate of reaction between different time interval is interval, z Rate =| 0- 100s [3.50 «10°0 - 4.00 10°] Rate = - spars 700 100-200 5 [3.00 100 -3.50x 10°] — Rate = > = 6Pais 200-3005 2.50% 1070 - 3.0010" ] Rate = a sPals As rate remains constant therefore, reaction is of zero order. i, Sarvapriya Vihar, New Delhi 11007 ‘webotte: wow fltfe.com "Fh 46106000, Fax 26519947 Chemical kinetics ISP-BR & DH-2021-C-XI-CH-CK-40- Exercise 10: To show that the reaction is first order, we calculate k at different interval of time using 2.303, (RI. P k= 5 TSloa pap” comes outto be same then proves that orders one Volume of KMnO, required at any stage corresponding to the amount of H:O2 [Rho and Vt = (RI. At 10 min 2308, (22.8 kyy = 2303 jog{ 228) - 5.02107 min 0 a too 222) 5.02107 min At 20 min 2.303), (228 ky = FS 8tog{ 222) - 5.0510" min 20 F3) " AAs the value of kis almost same, the reaction is of first order. Exercise 11: "Rate of reaction] Rate constant @ [itis defined as the change in| () | is defined as the rate of concentration of the reactant or chemical reaction when the | product with time, each divided concentration of each reactant | by its stoichimetric coefficient. appearing in the rate equation i taken as unit _ ( [i always has a unit of | (i) | Its units depends on the order of (cono)itime. a reaction. For n” order reaction | the units of rate constant = (conc)"* time” Gi) [depends on the It is independent of the | Concentration of the reactants. concentration of the reactants. | Exercise 12: The order of a reaction is defined as the sum of the exponents to which each concentration terms is raised in the experimently derived rate equation. For example in the reaction A+B + C—> Products and the rate law expression is r=K(Ay [8]'[C] Then, overall order of reaction = x + y +z Order of a reaction can be fractional value. An example of a fractional reaction is gas-phase decomposition of CHsCHO. CH,CHO,,, —>CO,,, +CH,,, and r= k[CH,CHO]” Exercise 13: "Order ofa reaction Moletularity of a reaction @ [iis sum of the exponents to | (@ ] In simple reactions, it is equal fo which the concentration terms in the number of molecules of the | | the rate law expression are reactants while in complex | raised to express observe rate reactions, it is number of of reaction molecules involved in rate determining step. (i) [it may be whole number, | (i) | Always a whole number. factional, zero or negative} | integer. _ Gi) [It can only be experimentally | (i) | Tt can be obtained by adding th determined. molecules of the elementary reaction. _ () | Wis for the overall reaction and | (jv) | Overall molecularly ofa no separate steps are written to complex reaction has _no ‘Wile Led, PITUEE House, 20-4, Kalu Sarat, Sarvapriga Vihar, New Delhi 110016, Ph 46106000, Fax 20ST 3942 ‘website: wu fgjee.com Chemical kinetics ISP-BR & DH-2021-C-XIL-CH-CK-41 obtain it. Significance; only slowest step is | significant (@ _| Tedoes not give an idea of the | (¥) | It explains the mechanism of the mechanism of the reaction but reaction. gives idea of kinetics of reaction. - Exercise 14: [Chemical Reactions ‘Nuclear Reactions (@ [This involves making and | (j) | This involves the conversion of breaking of bonds in order to one element into another rearrange the atoms (| Only electrons in the atomic | (i) | iF involves protons, neutrons, | orbitals are involved in the electrons, and other elementary | making and breaking of bonds. particles. (i) [A involves the absorption of or | (ii) | There reactions, are release of relatively small accompanied by absorption or amounts release energy. release of tremendous amounts of energ ()_| Rates of reaction are influences | (jv) | Rates of reaction normally are by temperature, pressure, not affected by temperature, concentrations and catalyst pressure, and catalyst 2._& |E-T Exercise 15: : xa A | 8710 ;E, = 103KJ mole Ty, = 273K, T= 293K kK, , 10320 00 ToT = 7.8710 "FITUEE House, 294, Kalu Sarat, Sareapriva V0 "Rew Delnt 2.303 x 6.314 293x273 x10" TTo0r6, Ph 46106000, Fax 26S13942 ‘website: waw fltjee.com Chemical kinetics ISP-BR & DH-2021-C-XI-CHCK-42 SOLVED PROBLEMS Subjective: ‘0' LEVEL. Pro61. What will be the initial rate of a reaction, if its rate constant is 10° min’ and the ‘concentration of reactant is 0.2 mol dm~? How much of reactant will be converted into product in 200 minutes? Sol. Reaction in Ist order as unit is min-’ of constant. K{conc]" 10° x (0.2]' = 2x 10 mol dm min* 2.303, a K= Slog Let a= 100 2.308), 100 200 9 100-x X= 18.12% x Prdb 2. The rate of reaction triples when temperature changes from 20°C to 50°C. Calculate ‘energy of activation for the reaction (R = 8.314 JK“"‘mole”') ‘Sol. The Arrhenius equation is K€ {T-T] K, Rx2.303| TT, | R= 8.314 JK ‘mole T, =20+273 = 293° Given: 62 and T: = 50 + 273 = 923°K Substituting the given values in the Arhenius equation, 0, gE, 323-293 No" * 2.308 x8.314| 323x293 g, = 2:303%8.314%323 x 203%0.477 . 30 E, = 28.81 KJ mole Prob 3. In Arhenius equation for a certain reaction, the value of A and E, (activation energy) are 4x10" sec’ and 98.6 KJ mole" respectively. At what temperature, the reaction will have specific rate constant 1.1% 10" sec” Sol. According to Arhenius equation Ke Aer oF logek = log, A ~ Flog, € E E, OF 2.80310g,,K = 2.303109, A - 5% oF logwk = 10g. A ~ saps 5314 2,308x16.56 T= 310.96°K 329A, Kalu Sarat, Sareapriga Vihar, New DelRt T1001 ‘website: waw fitjec.com ce raIOaIEkInetics ISP-BR & DH-2021-C-XIL-CH-CK-43 Prob 4. The energy of activation for a reaction is 100 KJ mol’ Presence of a catalyst lowers Sol, the energy of activation by 75%. What will be effect on rate of reaction at 20°C, other things being equal? The Arhenius equation is K= Ae ei In absence of catalyst, K, = Ae’ In presence of catalyst, K7 = Ae™"*" K Kk, 75 75 so, 2-0" or 2303102 = 75 or 2.303 — OK, or OK “arf 2803100 = Fata 107x203 75 K, — 2.303%8.314x293x10° ~ K, {As the things being equal in presence or absence of a catalyst, log 34x10" Ko must be = [2t2 in presence of catalyst K, rate in absence of catalyst = 2.3410" mK A|R ——————————————————————— EE ‘ie Led, PITUEE House, 294, Kalu Sarat, Sarcapriya Vihar, New Delhi 110016, Ph 46106000, Fax 26513942 ‘website: waw fjee.com Chemical kinetics ISP-BR & DH-2021-C-XIL-CH-CK-44- LEVEL -1 Prob 1. Thermal decomposition of a compound is of the first order. If 50% of a sample of the compound is decomposed in 120 minutes. How long will it take for 90% of compound to decompose? Sol. Half life of reacti We known that 0.693 _ 0.693 K = = = =5.77«10" min” tye 120 x1 mi Applying first order reaction equation, t= 2383 9g, 2 ita = 100, K Sax =90 or (a-x)= 2.303 2.303 577107 80°" 577.407 Prob 2. Catalytic decomposition of nitrous oxide by gold at 900°C at an initial pressure of 200 mm was 50% in 53 minutes and 73% in 100 minutes. (a) what is order of reaction? (0) how much it will decompose in 100 minutes at the same temperature but an initial pressure of 600 mm? so t= 399 min Sol. (a) Using first order kinetics equation and substituting given values, 2.303 200 In first case: K = 2208 jqq,, — 200 _ " 53%" 200-100 2.303 200 In second case: K = 2293 jog, 200 9.0131 min n second cas 100 °9° 200-146 K is same in both cases, so reaction is of fist order. (©) In first order reaction, the time required for the completion of same fraction is independent of initial concentration, the percentage decomposition in 100 minutes when the initial pressure is 600 mm will also be 73%. =0.0131 min Prob 3. The gas phase decomposition 2N,0, —>4NO, +0, follows the first order rate law. At a given temperature the rate constant of the reaction is7.5x10" sec”! The initial pressure of NzOs is 0.1 atm. (9 Calculate the time of decomposition of NzOs So that the total pressure becomes 0.15 atm. (ii) What will be total pressure after 150 seconds? Sol. _()The decomposition reaction is represented by 2N,0, —>4NO, +0, o wa Initial p, o 0 Aftertimet P,-2x 4x x Total pressure = pp 2x + 4x+x= py + 9x=0.15 atm or 3x = 0.15 ~0.10 = 0.05, = 295 - 9.0166 atm 3 Pressure of N,Os after t seconds, i, Sareapriya Vihar, New Deiht 110016, Ph 46106000, Fax 26ST0902 ‘webotte: wow fitjec com ie Remi caiihineiice ISP-BR & DH-2021-C-XILCH-CK-45 >, = 20.0166 = (0.1- 0.0332) = 0.067 atm Now applying first order rate law 2.303, Py 1 = 2308 ig, fo P = 2.808 9g 75x10" ~"° 0.067 (i) Afier 180 second, 2.303) 0.4 0.4 _ 1807.5 «10° 150 =F slog OF logge 21 - 0.0324 06 = 53.4 seconds 4885 Po - 2x = P = 0.0324 2x = (0.1 - 0.0324) = 0.06757= x= 0.0337 Total pressure = Po + 3x = 0.1 + 0.1012 = 0.201 atm Prob 4. Ata certain temperature the half life for the catalytic decomposition of ammonia were found as follows: Pressure (Pascals) 6667 13333 26666 Half lite period (hrs) 3.52 1.92 1.0 Calculate order of reaction. (twa), ( (tua)e From the given data, 3.62 _(13333)"" ta eeer) =" Sol. ) where n is order of reaction. in 1)log2= 0.3010«(n-1) n=2 Thus order of reaction is 2. 3b 5. If a reaction A -» P, the concentration of reactant A are Co, aCo, a°Co, a°Co. after time interval 0, t, 2t, 3t where a is constant. Given 0 < a < 1. Show that the reaction is of ist order. Sol. = 2308 jog_@ tax Ift=t, a= Co and a—x= aCo (= 2:208j9q 00. _ 2.808451 t aco t a Ift=2t, a= Co and (a~x) = a°Co logo a’Co teat, K = 233 j9g Co 3t a’Co Kis almost same, so reaction is Ist order. FITIEE House, 294, Ratu Sarat, Sarvapriya Vihar, New DelN 110016, Ph 46106000, Fax 20513992 ‘website: war fljce.com Chemical kinetics ISP-BR & DH-2021-C-XIL-CH-CK-46- Ra* emits a- particle to produce radon gas seRn* haltsife period of both are respectively 1620 years and 3.8 days, calculate (a) Volume of Radon in equilibrium with 0.226 g radium at 27°C and (b) Decay constant 2 of Ra Prob Nx0.226__N 226 1000 Sol. (a) No of atoms in 0,226 g Ra = New (trades (traden 3.8xN Naa = aan 2SxN OF Nee * 7620965 «1000 N atoms are present in 222 9 of Rn 3.8222 teq= 28222 _ wt of Rn gas at ea = +9 65. 10009 at NTP 222 g Ra occupy 22400 cc volume 3.8222 Volume of —3:8%222__ NTP clume of +559,.365. 10009 FR" 22400 3.8222 222,” 1620 365% 1000 2243.8 1620« 36510 Py = 1 atmT)=273°C P) = 1 atm T; = 273+27=300 K 22438 "7620365 x10 1122438 300 5 22498 __ 300, = 1 581.10 620x365%10%073° 1 “ 12 = 3.8 days) (©) Decay constant of Ri 0.693 38 182 per day "76106000, Fax 26819900 Chemical kinetics ISP-BR 8 DH-2021-C-XILCH-CK.47 LEVEL -11 Prob 1. For the following 1 order parallel chain reaction 58 kK, aw ec given that K, = 5x10° s & Kz = 610° s" then find out half life of A. Sol. For parallel chain reaction Koverat = Ky + Ko = 5:10°+ 6x10°s" = 56x10°s? = 0.693 56.107 = 0.69310" 69.310" sO ta = 1.24210" Sec 1.24210" Sec Prd 2. $$Cu (half ite = 12.8 h) decays by emission (38%), emission (19%) and &* capture (43%). Calculate partial half - lives for each of the decay process. aot Ben Sol. aN Let the rate constant of the above emission process be ky, k & ks respectively and the overall rate constant be K. Then K= Ket Ke + Ky = 2588 _ 9683, te 128 0.693 4 = 0,38« 288; 128" 0.693 «12.8 = SoS TE = 33.68 h 038-0693 Similarly t;= 289%. 0.683 _ 0.699 49 g=67.36n K, 0.19K ~ 0.19x0.693 693 0.693128 12.8. 99 76h 43K 0.43x0.0693 0.43 ‘Where t,, t2 & ts are the partial half lives for respective emissions. A sample of river water was found to contain 8 x 10-'° tritium atoms, ;H per atom of ordinary hydrogen. The half life of radioactive tritium is 123 years. log 95.186=1.978) (a) What will be the ratio of the tritium to normal hydrogen after 49 years, if the sample is stored in a place where additional tritium atoms cannot be formed? (b) How many tritium atoms would 10 gram of such sample contain 40 years after initial sampling? Wetiee Led, PITUEE House, 294, alu Sarat, Sarvapriva Vihar, New Delht 110016, ‘website: wa fljee.com Chemical kinetics ISP-BR 6 DH-2021-C-XII-CH-CK-48————___—"—_" Sol. (a) _49.years is exactly four half life's a —' as ty & te, te, Be 16 After 49 years, Number of ttium atom = Tx ttm inital 1 7 9 AB 102 5x10 ie Ratio= NH = 5+10 NH 4 O) MyoF 2 mol Hatom = 12 mol = Av\NO= 6.7 « 10% atoms NO. of tritium atoms =8 x 10°" x 6.7 x 10% 2.303 |. N, 2.303 jog Ne tN 693 _ 2.903 ,N, _ 2303, N, 123y. N 40y. N ‘on solving, = 106 N, After 40 years, number of tritium atom left = 106 x 5 x 10° = 0.530 x 10° = 5.3 x 10° atom Prob 4. The hat ite of radon is 3.8 days, Calculate the time in which its 1th of the amount will be left behind, (Log 2= 0.3010) sor 0.693 th Se per da = 0.182 per day Let the intial amount (N) be a then Ne= a/20 2.303 ,,0No x ON, 2.303, 2 _ 2.303 2.303 gq 2 _ 2:308),499 «16.45 a 0.182 3726 ~ 0.162.°970 = 16-45 day 8 oS So Let kicks = 1:16, Calculate the ratio, Cl atthe end of one hour (A) A Se assuming that ky =x hr" c na] Sol. Ae (+4, IA] AMAT fey at fo” Integrating with in the required limits, we get Tite Led, FUTUEE Rouse, 25 ——______Chemical kinetics _isp.pp & DH.2021-CXILCH.CK-49 inlAl = (ky + hot tl (ALslol-fe) . In ELE) = (kitka)t since St SI i - tay Gete) TAL fete" -9 In 16x Prob 6. How much time would be required for the B fo reach maximum concentration for the reaction? A Given k, = M2: 4 Sol, t= 4 minutes Prob 7. ky = xh ky ky = 1:10 Calculate [C/{A] after one hour from the start of reaction. Assuming only A was present in the beginning. [Cl 10 erm _ sot age) Prob 8. Ac™” has a half life of 22 years wrt radioactive decay. The decay follows two parallel paths, one leading the Th” & the other leading to Fr“? The percentage yields of these two daughter nuclides are 2% & 98% respectively. What is the rate constant in year for each of the separate paths? Sol 6.3.x 10 yr 3.087 x 1077 yr! 25a, Kalu Sarat, Sarvapriya W ‘webatte: wow fe "Rew Delhi 110016, Ph 46106000, Fax 26510942 Chemical kinetics ISP-BR & DH-2021-C-XIL-CH-CK-50——___——"""" SAS UES 1-0 vu Objective: True and False: Prob 1. Sol. Prob 2. Sol. Prob 3. Sol. Prob 4, Sol. Prob 5. Sol. Prob 6, Sol. Rate of a reaction depends only on the number of collision per unit volume per unit time. False The minimum amount of energy which the colliding molecules must possess is known a8 energy of activation False When an a-particle emits from a radioactive element, its mass number decreases by two unit and atomic number decreases by four unit. False The hydrolysis of an ester is second order reaction because it involves both water and ester in rate determining step. False For elementary reaction, order is same as molecularity. True Electron capture occurs with the nuclei lying below the stability belt, in which an electron from the K-shell is captured by the nucleus. True Fill In the Blanks: Prob 7. Sol. Prob 8. Sol. Prob 9. Sol. Wariee Led,, PITUEE House, 297 The difference between the activation energies of the forward and backward reaction respectively is known as of the reaction. Enthalpy In a emission, the resulting element has a mass number less by units and atomic number by units. Four, Two In electron capture, the n/p ratio. Increases Fax 26515942 chem icsiiiinetics ISP-BR & DH-2021-C-XIL-CH-CK.51 LEVEL -1 MCQ Single Correct: Prob 1. Sol. Prob 2. Sol. Prob 3. Sol. Prob 4. Sol, "Wie Led, FITUBE House, 25, For the reaction 44 + B ——» 2C + 2D, which of the following statement is not correct? (A) the rate of disappearance of B is one fourth of the rate of disappearance of A (8) the rate of formation of C is one ~ half of the rate of consumption of A (C) the rate of appearance of D is haif of the rate of disappearance of B. (0) the rate of formation of C and D are equal. The rate of appearance of D is double the rate of disappearance of B. (c) When concentration of reactant in reaction A —» B is increased by 8 times, the rate increases only 2 times. The order of reaction would be (Ay2 (8) 173 (C4 (0) 172 (8) (i) r= ka” (id) 2r = k(Ba)" dividing (i) by (i), 2= 8" or 2° = 2! v3 For the following first order reaction A —» products, which one of the following is correct plot of log(A] versus time? A (8) q ; (c) -) g| Z| ql 3 (B). Plot of log(A) vs time is finear with (-ve) slope. For the reaction A, B, C the order of activation energy is ac 7 (A)A>B>C (8)B>C>A (C)C>B>A (0)AC (C). On the basis of slope (-3) Sarat 5 Vihar, New Delhi 11001 sarvapriy ‘webaite: | PR 46106000, Fax 26519942 Chemical kinetics ISP-BR & DH-202 1-C-XI-CH-CK-52————_——"""_SUSUIGS Prob 5. The rate constant of a reaction increases with increases of temperature because (A) the activation energy increases (B) the population of activated molecules increases (C) the activation energy decreases (0) the population of activated molecules decreases Sol —_ With increase in temperature, population of activated molecules increases. (B) ‘Sarat, Sarvapriya Vihar, New Delht 110016, ‘website: war fljee.com ‘tie Lea, PUTUEE kinetion -ISP-BR & DH-2021-C-XII-CH-CK-53, LEVEL -U MCQ Single Correct: Prob 1. Sol. Prob 2. Sol. Prob 3. Sol, Prob 4. Sol. In the formation of sulphur trioxide by contact process, 280, +O, —»2S0O,, the rate -d[O, of reaction was measured as =e) = 2.510 mole lit'sec The rate of appearance of SO; in reaction will be (A) -5x10* mole lit’ sec’ (B) -1.25«10 mole lit sec’ (C) 2.510" mole lit ‘sec (D) §.0x10 mole lit’ sec [0,] _ 4[S0,] dt 2dt (0) The rate of gaseous reaction is equal to K(AI[B]. The volume of the reaction vessel containing these gases is suddenly reduced to one — fourth of the initial volume. The rate of reaction would be (A) 116 (B) 16/1 (c) 18 (0) a1 {Initially rate = K[A][B] When volume of the vessel is reduced to one - fourth, concentrations becomes 4 times, hence new rate = = K(4a)(4b) = 16kab i.e. it becomes 16 times. (B) If initial concentration is reduced to 1/4” in a zero order reaction, the time taken for half the reaction to complete (A) remains same (8) becomes 4 times (C) becomes one = fourth (0) doubles A For zero order reactions, ty = [Ael ‘When [Ad] is reduced to 1/4", ty will become 1/4” (c) The half ~ life of a reaction is halved as the initial concentration of the reactant is doubled. The order of reaction is (aos (1 (2 (0 1 (tvs), [Ach STAT Gals [Aol [Acl, sear tye) ose mala torn=2 or(n- 4 (c) Chemical kinetics ISP-BR & DH-2021-C-XIL-CH-CK.54——___" “ISS” NARGIS Prob 5. Enzyme catalysed reaction is faster than metal catalysed reaction because its activation energy is (A) greater (8) lower (C) same (0) none of these Sol, Activation energy is lower. e) MCQ Multiple Correct Type: Prob 6. For the reaction, 3BrO” —+Br0; +2Br in an aqueous alkaline medium at 80°C, the i =o] tet value of the rate constant in the rate law in terms of —-=— is 0.0561 mof's d[Bro, at wil be the rate constant when the rate lw is stated in tors ot (L222 (A) 18.7x10°Lmor's™ (8) 37.4<10°Lmol's (C) 0.0187Lmor's" (0) 18.7x10"Lmol's So, A.C Prob7. The reaction rate of the reaction Hg) + Brg) ——> 2HBr(g) is given by Rate = KHZ) [Br2)” Which of the following statements is/are true? (A) Order of reaction is 2 (B) Molecularity of the slow step of reaction is 2 (C) Order of reaction is 1.5 (D) Molecularly of reaction is 1.5 Sol BC Prob 8. The decomposition of a substance follows first order kinetics. Its concentration is reduced to 1/8” of its initial value in 24 minutes. The rate constant of the decomposition process is/are (A) 1/24 min” (B) 0.692/24 min" (C) (2.303/24) log (1/8) mins" (O) (2.303724) log (8/1) min’ So. Assertion ~ Reason Type: The following questions consist of two statements each printed as Assertion and Reason. While answering these questions, you are required to choose any one of the following four responses. (A) If both Assertion and Reason are true and the Reason is a correct explanation of the Assertion, (8) If both Assertion and Reason are true but Reason is not @ correct explanation of the Assertion (C) If Assertion is true but the Reason is false. (0) Ifthe Assertion is false but the Reason is true. Prob 9. Assertion: Positive catalysts increase the rate of reaction Reason-2: Catalysts decrease the value of \G’ Sok rreapriya Vihar, New Delnt 1007 ‘website: wun fljee.com "Ph 46106000, Fax 20819000 ———_Chemical kinetics isp pr a pH.2021.C-XN.CHCK-SS Prob 10. Assertion: If the activation energy of a reaction is zero, temperature will have no effect on the rate constant Reason: Lower the activation energy, faster is the reaction. Sol. B Prob 11. Assertion: The order of a reaction can have fractional value. Reason: Order of a reaction cannot be written from balanced chemical equation Sol. B Comprehension Based Questions: ‘Comprehension -| Yukawa suggested that a fundamental particle called mesons or pi-mesons (pions) exist in the nucleus. These particles oscillates between the neighbouring nucleons with velocity close to that of light. These may be neutral, positive or negative electrically Prob 12. Proton can change to neutron by accepting (A) negative pion (8) +ve pion (C) neutral pion (O) Pparticle So. A Prob 13. Neutron can change into proton by accepting (A) neutral pion (8) ~ve pion (C) +ve pion (0) e-particie Sol C Comprehension ~ Il Dinitrogen pentoxide decomposes to NO; and O; following first order kinetics N,0, (g) + 2NO, +0,(g) 0.2 mole of NzO; was taken in 2 L vessel and heated at 200 K. The concentration of N:Os is ‘measured at different intervals and the following graph was plotted log (N,O,.)—> Time ——> Prob 14. Slope of straight line in graph ‘A’ is -1.2 x 10° sec™' what is haif life of the reaction (A)2.5x 107 5 (B) 25x10" s (C) 12.5 x 10% (OD) 25x10" m Sol. B is Sarai, Sarvapriya Vihar, Rew Delhi "110016, Ph 46106000, Fax 26513902 ‘website: wu fljce.com Chemical kinetics ISP-BR & DH-2021-C-XILCHCK-56- Prob 15. Calculate the rate of reaction after 5 x 10° s (A) 6.84 mol L's (8) 3.42 mol L's (C) 6.84 mol L* (D) 6.84 mol s* Sol. A Comprehension — Itt tye for n!” order reaction. If a reaction nA -+ Products aaa] then rat at “Klar Pe Integrating both sides and doing further reaction We get tye = ges Where ao = initial conc. of A. Hence ty, Prob 16. For a reaction of n” order the ratio of is a function of (A) a0 (B) kn (on wot Sol. c Prob 17. The following data was obtained for a chemical reaction [ao(movam™y 05 [1.1 2.48 tt 4280 _| 885 174 The order of the reaction is (At (B) 2 ()3 (D4 Sol. c Matrix Match type Question: Prob 18. Match the List — | with List ~ i: 7 tist=T E ao List= if (Reaction) i ype) (P) | 1" order (1) | Unit is mot L's? (Q) | 2” order (2) | tia = ta (R)_| 3 order (3) | Unit is x7 (S) | 0” order 4 | tax ve PQ sR Ss A 1 2 3 4 @® 3 2 4 7 © 4 3 2 474 0 2 4 4 3 Sol. B "Hew Delht 110016, Ph 46106000, Fax 26513902 ‘website: wun fltjee.com eB StiieaiEinetics ISP-BR & DH-2021-C-XI-CH-CK.S7 Prob 19. Match the Column ~ | and Column = I, Column -T i Column = it | A 3 (p) | 99% completion tae 2 xtyo (8) | t= 2 xtp (@) | Zero order — (C) | 6.909% ()_| First order _ (OD) | 2.303% (8) | 90% completion i) Sol. (A) > (4, 8); (B) > (95 (C) > (p, 1); (D) — (r, 8) Answers Q.20, Q21 and Q.22 by appropriately matching the information given in the three columns of the following table. Column-1, 2 and 3 contain order of reaction, half life and unit of rate constant respectively. ((2’is initial concentration of reactant) { Column-1 Column-2 Column-3 ] () 1 order (W) tin a (p) mol litre" seo* 2 order (09) trax Wa (q) ser" 1 (i 3” order tin =e () mol Titre see | (iv) zero order (2) tia independent of a (3) mol "ttre sec Prob 20. For the decomposition of HI on the surface of gold, the only correct combination is (A) (iv), (w), (p) (B) (i), (), (Q) (©) (il, (w), (p) (D) (), (2), (P) Sol. A Prob 21. For the decomposition of N,Os, the only correct combination is (A) (iv), (), (p) (B) (i), (2), (9) (C) (), (W), () (0) (), (y), (e) Sol. B Prob 22. For the reaction: NHNO, —>N, +2H,O The only correct combination is A) (i), 0) (B) (ii), (w), (p) (C) (i), (W), (s) (O) (i), (2), Sol. D () Decomposition of HI on the surface of gold is a zero order reaction Gi) Decomposition of NzOs is a 1* order reaction. (i). Hydrolysis of ester is a 1* order reaction Soa SSS ——— TWH Led, PUTUBE House, 29-4, Kalu Sarat, Sarvapriva Vihar, New DelN 110016, Ph 46106000, Fax 26519902 ‘website: wun flgjee.com ISP-BR & DH-2021-C-XI-CHCK-SB- Chemical kinetics Numerical Based Question/Decimal Based Questions: Prob 23. Sol, Prob 24, Sol, Prob 25. Sol. Wa Le The decomposition of methyl iodide 2CH,I(g) > CH, (9) +12(9) at 273°C has a rate constant of 2.418 » 10° s“' If activation energy for the reaction is 179.38 KJmol' what is the value of collision factor ‘A’ at 273°C? If it is given as x « 10"? s”, then find x. flog 2.418 = 0.3834] 35 The ratio *, for a first order reaction would be equal to 3 G.M. Counter is used to measure the activity of a radioactive sample. in the absence of a radioisotope, it records 3 dps, but in the presence of radioisotope, it first records 23 dps, and after 10 min 13 dps; what will be its reading (in dps) after 20 min? ‘PITURE House, 29-4, Kalu Sarat, Sarvapriva Vihar, New Delhi 110016, Ph 46106000, Fax 65100 website: wa flfee.com je hem icalEinetics ISP-BR & DH-2021-C-XIL-CH-CK-59 CHAPTER PRACTICE PROBLEMS (CPP) Subjective: LEVEL -1 ea, 1. For reaction A —> B, the rate constant k, = A,e“*"""! and for the reaction X —+Y, the rate constant kj= A,€" then find out the temperature at which k; 10° and Ea;= 600 cal / mol, Ea;=1800 cal/mol, 1m: R=2cal/K-mol) 2. For first order paralled reaction K, and Kz are 4 and 2 min’ respectively at 300 K. If the activation energies for the formation of 8 and C are respectively 30,000 and 38,314 joule/ mol respectively. Find the temperature at which B and C will be obtained in equimolar ratio 3. Carbon monoxide reacts with O2 to form COz; 2CO(g) + O2(g) ——> 2CO;(g) information on this reaction given in the table below. [CO] mou. | [Oa] movi. Rate of reaction (mov. min) 0.02 0.02 410" 0.04 0.02 16x10 * 0.02 0.04 8x10" What is the value for the rate constant for the reaction in proper related unit? LEVEL -1 4. Half-life for the zero order reaction, A(g) —> B(g) + C(Q) and half life for the first order reaction X(g) —> Y(g) are equal. if completions time for the zero order reaction is 13.86 min, then calculate the rate constant (in hr") for the reaction X(g) + Y(Q). 5. Anelement > X, which is radioactive and has half life tyz= 20 days. The daughter nuclei formed after decay of Xis; ¢ Y Calculate the volume of He collected at S.T-P after 40 days if 2 mole of X is placed in a sealed tube sreapriga Vihar, New Delhi 110016, Ph 46106000, Fax 26519942 ‘website: war fljee.com Chemical kinetics ISP-BR & DH-2021-C-XIL-CH-CK-60- Objective: MCQ Single Correct Questions: 1. For an elementary reaction 24+B —> A,B if the volume of vessel is quickly reduced to half of it’s original volume then rate of reaction will be (A) unchanged (8) increase four times (©) increase eight times (O) decrease eight time 2. 99% of a first order reaction was completed in 32 minutes then 99.9% of the reaction will complete? (A) 50 min (8) 46 min (©) 48 min (0) 49 min 3. The decomposition of azo methane, at certain temperature according to the equation (CHs):.N>——> C2HetN> is a first order reaction After 40 minutes from the start, the total pressure developed is found to be 350 mm Hg in place of initial pressure 200 mm Hg of azo methane. The value of rate constant K is: (A) 2.8810" sec” (8) 1.25x10 sec’ (©) §.77%10°* sec" () none of these 4 karBet0" min" ogy A decomposes as A(a) 4 the rate of appearance of B, taking 2M atom? C8) concentration of A, is equal to: (A) 2x10°Ms" (@) 410° M (C) 810° Ms! (0) none of these 5. For an slow exothermic chemical process occurring in two steps as follows (A+B 25 x (iy X > a ‘The process of reaction can be best describe by a PE (A) . 8) A) JA+B * AB ® AB AB (Reaction coordinate) Re RC ike PE M >» PE © ©) AB ANB AB ASB L__x _s, RC. RC. "GBA, Kalu Sarat, Sarvapriya Vihar, New Delhi 110016, Ph 46106000, Fax 20813902 ‘website: wun fljee.com chem icsiikinetics -ISP-BR & DH-2021-C-XILCH-CK.61 MCQ Multiple Correct Questions: 6. Which of the following reactions will have fractional order for Az or 82? (A) A, 2 A+A(Iast), A+B, —>AB+B(slow); A+B = AB(fast) (8) 8, =8+B(fast); A, +B —+AB+A(slow), AB — Product (fast) (©) A, —+C(slow); C +B, = D(fast):D +A, = Product (©) All have fractional order 7. In an experiment on two radioactive isotopes of an element (which do not decay into each other), their molar ratio at a given instant is 3. The rapidly decaying isotope has larger mass and an initial activity of 1.0 4Ci. The half lives of the two isotopes are 12 and 16 hrs respectively. Now identify the correct statement(s) among the following (A) the activity of one of the isotopes is 0.0626 u Ci after two days (B) the activity of another isotope is 0.0313 u Ci after two days. (C) the molar ratio of first to that of second isotope after two days is 1.0 (D) the molar ratio of first to that of second isotope after two days is 1.5 8. The table give below gives the kinetic data for the reaction OCI +f -> Ol +C at 28 K Expt. focty 17 TOH] +aLIo- No motellitre molefitre | moleflitre “ALC J potattres8e" 1 0.0017 0.0017 1.0 1.75 x10 2 0.0034 0.0017 1.0 3.50 x 107 3 0.0017 0.0034 10 3.50 x10" 4 0.0017, 0.0017 05 3.50 x 10" Which are correct statements? (A) The rate law for the reaction is rate = K[OCr II YOH] K[oer Jr] (8) The rate law for the reaction is rate = —-——_ [ow] (©) The overall order of the reaction is 1 (D) The unit of rate is mole litre™'sec™* Numerical Based Questions: 9. The rate of a reaction gets double when the temperature changes from 7°C to 17°C. By what factor will it change when the temperature change from 17°C to 27°C. 10. At 500 K, the half life period of a gaseous reaction at an initial pressure of 80 KPa is 350 sec. ‘When the pressure is 40 KPa, the half life period is 175 sec the order of the reaction is THE Ltd., PITIEE House, 29-4, Kalu Sarat, Sarvapriga Vihar, New DelNt 110016, Ph #6106000, Fax 26513942 ‘website: wun fljee.com Chemical kinetics ISP-BR & DH-2021-C-XIL-CH-CK-62- ASSIGNMENT PROBLEMS Subjective: ‘0’ LEVEL, 4. The rate of decomposition of dimethyl ether to form CHs, Hz and CO in a closed vessel is expressed as, Rate =K(CH,OCH,)"" or K(Poxcou) If the pressure measured in bar and time in minutes, then what are the units of rate and rate constant? 2. A reaction is first order in A and second order in B. () write differential rate equation. (ji) how is the rate affected when (a) concentration of B is tripled (b) concentration of A and 8 is doubled? 3. Nitric oxide, NO reacts with oxygen to produce nitrogen dioxide: 2NO+ 0, —>2NO3 What is the predicted rate law, if the mechanism is NO +0, ==NO, (fast) and NO, +NO="=NO, +NO, (slow) 4. For a homogenous decomposition of N2Os into NO; and O2 2N20 5,5)? 4NO, 5) + Orig, ~4a[N,0,] rate = -2_ "= K[N,O,] At Find out the order with respect to [N2Os] 5. Can a reaction have zero activation energy? 6. State any one condition under which a bimolecular reaction may be kinetically of first order? 7. What do you understand by the rate of a reaction? How is it expressed? How is the rate of reaction determined? 8. What do you understand by order of a reaction? How does rate law differ from law of mass action? Give two examples each of the reactions of () zero order (i) first order. 9. List the main points of difference between order and molecularity of a reaction. What do you ‘mean by the mechanism of a reaction. Explain taking at least two examples. 10. Derive the equation for rate constant for a first order reaction. What would be the units of the first order rate constant if the concentration is expressed in moles per litre and time in seconds? Wiles Lea, FUTIEE House, 29-4, Kalu Sarat, Sarcapriga Vihar, New Delhi 110016, Ph 46106000, Fax 20813942 ‘website: wu flgjee.com jenemics kinetics ISP-BR & DH-2021-C-XILCH.CK-63 11. Write short notes on (i) fast reactions (ji) photochemical reactions. 12. Write the rate law for a first order reaction and justify the statement that half life of such a reaction is independent of the initial concentration of the reactants? 13. The reaction 2H, +O, —+2H,O is feasible. How is that hydrogen and oxygen mixture allowed to stand at room temperature shows no formation of water at all? 14, What are pseudounimolecular reactions? Explain the help of suitable example. 15. What is activation energy? How is the rate constant of a reaction related to its activation energy? 16. The rate law for the reaction, 2Cl,0—+ 2CI, + 0, at 200°C is found to be rate = K[C/,O) (a)How would the rate change, if [Cl:0] is reduced to one third of its original value? (b)How should the [Cl,0] be charged in order to double the rate? (c)How would the rate change, if (Cl,0] is raised to three fold of its original value? 17. Explain in brief the collision theory of reaction rates. 18. What is the difference between °,e and °,8? 19. Complete the nuclear reaction: @ 4H $0. ii), SMn(n,y). 20. Explain the following: () Mass defect (i) Binding energy 21. For the reaction A +2B——+2AB,,, the rate constant is 1.26 x 10° L mol 's’ What is the order of the reaction? 22. Give any one example of (zero order reaction (ii) first order reaction 23. The tyzof a first order reaction is 60 minutes. What percentage will be left after 240 minutes? 24, Which particle emission produces isobars? "New Delhi 1100I6, Ph 46106000, Fax 26513942 Chemical kinetics ISP-BR & DH-2021-C-XILCH-CK-64 LeveL -1 1. Forthe following reaction 24 +B +2C+A,B + 2C the rate law has been determined to be R= KAFIB] where K =3x 10° moll? seo" For this reaction the initial concentration of various species. [A] = 0.1 motel” [B] = 0.2 mole it, [C] = 0.8 mole” Determine the rate after 0.04 mole lit"' of A has reacted. 2. For the reaction 2A + B —+ C, the rate of formation of C is 0.25 mole lit' hr’ What is the rate of disappearance of A and B? 3. The rate of a particular reaction doubles when temperature changes from 27°C to 37°C. Calculate the energy of activation for such reaction (R = 8.314 JK” mol’) |. For a chemical reaction the energy of activation is 85 KJ mol’ If the frequency factor is 4.0 10°mole“lit s* what is rate constant at 400 K? 5. Calculate the ratio of the catalysed and uncatalysed rate constant at 20°C if the energy of activation of a catalysed reaction is 20 KJ mol and for the uncatalysed reaction is 75 KS mot’ 6. For a first order reaction, the rate constant is 0.1 sec’’ How much time will it take to reduce the concentration from initial value of 0.6 mot” to 0.06 mol lit"'? 7. Forthe reaction, 2NO + 2H, —> Np + 2H,0 the following kinetic data were obtained. (Expt. No. [A] mole lit mole lit Rate of reaction 1 0.12 0.12 0.2510 2 0.12 0.24 1.010 3 0.24 0.24 2x10 Determine the rate law and rate constant, 8. In areaction A> B + C, the following data were obtained: tin seconds. 0 900 1800 cone. of A 50.8 19.7 7.62 Prove that itis a first order reaction. 9. A first order reaction is 20% completed in 10 minutes. Calculate (i) the specific rate constant of the reaction. (ii the time taken for the reaction to go to 75% completion 10. The halflife of a first order reaction is 10 seconds. Calculate the time for completion of 99.9% of the reaction 14. In the following reaction: 2H,O,——2H,0+0, Rate of formation of O2 is 3.6 Mmin™ (a) What is rate of formation of H,0? (b) What is rate of disappearance of H.02? ———————————— SS "Waite Led, PITUEE House, 29-4, Kalu Sarat, Saroapriya Vihar, New Delhi 110016, Ph 46106000, Fax 20819902 ‘webotte: wow fi eRe misaei netics: ISP-BR & DH-2021-C-XILCH-CK.65 12. In the following reaction, rate constant is 1.2 x 10? Ms‘ A > B, What is concentration of B after 10 & 20 min, if we start with 10 M of A? 13. A first order reaction is 75% completed in 72 mins. How long will it take for: (50% completion Gi) 87.5% completion 14, The half life period of decomposition of a compound is 50 minutes. If the initial concentration is halved, the half life period is reduced to 25 minutes. What is order of reaction? 15. The energy of activation of a first order reaction is 104.5 kJ mol” & pre exponential factor (A) is 5 « 10° sec’ At what temperature will the reaction have a half life of 1 minute? "Mite Led, FITUBE House, 294, Kalu Sarat, Sarvapriya Vihar, New Delhi 110016, Ph 46106000, Fax 26513942 ‘website: wow fitjee com Chemical kinetics ISP-BR & DH-2021-C-XI-CH-CK-66- LEVEL - 11 The first order reaction has K = 1.5x10*per second at 200°C. If the reaction is allowed to run for 10 hours at the same temperature, what percentage of the initial concentration would have changed into the product? What is the half life period of this reaction? 2. The catalytic decomposition of H202 was studied by titrating it at different intervals with KMnO, and the following data were obtained: tin seconds 0 600 1200 Volume of KMnO. (mi) 22.8 138 a3 Calculate the velocity constant for the reaction assuming it to be first order reaction. 3. The decomposition of AB,,—>A,,.+B,,, iS first order reaction with a rate constant K=4x10“s' at 318 K. If AB has 26664.5 Pa pressure at the initial stage, what be the pattial pressure of AB after half an hour? Lo 3s'< of a reaction is 1.82. Calculate the energy of activation in 4, Temperature coefficient = « calories. (R = 1.987 cal degree”! mole™') 5. Two reactions (i) A—> products, (ii) B —> products, follows first order kinetics. The rate of the reaction () is doubled when the temperature is raised from 300 K to 310 K. The half life for this reaction at 310 K is 30 minutes. At the same temperature B decomposes twice as fast as A. if the energy of activation for the reaction (i) is half that of reaction (i), calculate the rate constant of the reaction (i) at 300 K. 6. The inversion of cane sugar is studied by recording the angle of rotation of a plane of polarised light. in a typical experiment at 18°C, 20 mi of a 15% cane sugar solution was mixed with 2 ml of conc. HCI and the angle of rotation was recorded with passage of time and the following data were collected Time (min.) 0 5 10 15 2 Angle of rotation 17.2 167 162 15.7 -6.0 ‘Show that the inversion of cane sugar follows the first order kinetic and also determine the rate constant of inversion 7. The reaction, ‘Sucrose —> glucose + fructose Takes place at 308 K in 0.5 N HCI. At time zero the initial rotation of mixture is 32.4° After 10 ‘minutes the total rotation is 28.8° If the rotation of sucrose per mole is 85°, that of glucose is 74° and of fructose is -86.04°, calculate the rate constant of the reaction. The volume of the solution is maintained at 1 lit. 8. A radioactive element A says to B, “I am half of what you were and you are one fourth of what Iwas. More over, | was 1.414 times than what you were" If the half life of A is 8 days, what is half life of 8? 7 Wei Led, FITJEE House, 20-4, Kalu Sarat, Sarcapriya Vihar, New Delhi 110016, Ph #6106000, Fax 20513902 ‘website: ww fltfes.com Chemical kinetics 9. For a fist order reaction A —> B the dependence of rate constant K on temperature is given by logiok =23.02~ 8410 20 % solution of A by mass decomposes to the extent of 25% in 20 minutes at 298 K. What will be the percent dissociation of A in the same time at 340 K, if we start with its 30% solution? 10. The half life period of 3:°R, is 140 days. In how many days 1 gm of this isotope is reduced to 0.25 gms? 11, A> B+ where Tis in Kelvin, Time t Total pressure of (B+C)| _P; Pa Find K if itis a first order reaction. Wee Led, FITUEE Ho "Bo:A, Kalu Barat, Sarsapriya Vihar, New Delhi 110016, Ph #6106000, Fax 26513942 ‘webatte: wow fitfec.com ISP-BR & DH-2021-C-XIL-CH-CK-67 Chemical kinetics ISP-BR & DH-2021-C-XILCH-CK-68- ——————_—_—_—_————————— Objective: oo LEVEL -1 1. Which of the following is true for order of reaction? (A) Itis equal to the sum of exponents of the molar concentrations of the reactants in the rate equation (©) Itis always a whole number (C) Itis never zero (0) It can be determined theoretically 2. Time required for 100% completion of a zero order reaction is A) 2Kia (©) ark (©) alk (0) ak a(y) 2 3. The rate of reaction 2x + Y —> products is given by “7? =[X}'[¥]. if X is present in large excess, the order of the reaction is (A) zer0 ©) wo (©) one (0) three 4. The rate of a reaction at aferent times { &) is found as follows Time (in minutes) Rate (in mole L | 0 2.80107 10 2.78 x107 _ 20 2.81107 30 2.79107 The order of reaction is (A) zero (©) one (©) two (0) three 5. In Arhenius equation, the fraction of effective collision is given by (A (©) none of these 6. The plot of fog k versus 1/T is linear with a slope of (A) EUR (8) -E/R (©) £v2.303R (0) -€,/2.303R 7. Inthe reaction 2A + B —+ products, the order w.r. A is found to be one and w.r.t. B is equal to 2. Concentration of A is doubled and that of 8 is halved, the rate of reaction will be (A) doubled (8) halved (©) remains unaffected (©) four times “Waive Lid, FITUEE House, 29°, Kalu Sarat, Sarvapriva Vihar, New Delhi 110016, Ph 46106000, Fax 26519942 ‘website: wun fljce.com Chemical kinetics ISP-BR & DH-2021-C-XII-CH-CK-69 8. The unit of rate constant and that of rate of reaction are same for (A) first order ) zer0 order (C) second order (©) all are wrong 8. Agaseous reaction, A, (—+8(@)+4c(0) ‘Shows increase in pressure from 100 mm to 120 mm in § minutes. The rate of disappearance of Aris (A) 4mm min (8) 8mm min (C) 16 mm min (0) 2mm min" 10. A first order reaction is half — completed in 45 minutes. How long does it need for 99.9% of the reaction to be completed? (A) 20 hours (®) 10 hours, (C) 7.5 hours, (©) Shours 11. For the first order reaction, half life is 14 sec. The time required for the initial concentration to reduce to 1/8” of its value is (A) 28s (8) 42s (©) (14)? s (O) 145 12, The rate constant for the reaction2N,O,—>4NO, +0, is 2x10 sec’ if the rate is1.210*mol lit’ sec’, then concentration of NOs in mole L ' is 14 12 (©) 0.04 ©) 06 13. In the first order reaction the concentration of the reactants is reduced to 25% in one hour The halflife period for the reaction (A) 2 brs (©) 4hrs (©) V2 hrs (0) 1/4 rs. & 14. Which one is correct for K = Ae *"? (A) Ea is energy of activation (©) Ris Rydberg constant (©) Kis equilibrium constant (©) Ais adsorption factor 15. For a first order reaction Tysis 1200 seconds. The specific rate constant is (in sec ') (A) 10° (8) 107 (c) 10° (0) 10% 16. 7JAl isa stable isotope 72Al is expected to disintegrate by (A) «- emission (©) B- emission (©) positron emission (©) neutron emission 17. The number of « and fi particles lost when 32°U changes to 2°Pb are (A) 8.68 (8) 6,88 (C) 6,68 (0) 80,88 FINUEE House, 294, Ratu Sarat, Sareapriga Vihar, New Delhi 110016, Ph 46106000, Fax 6cTOO4T ‘website: ww fitjee.com Chemical kinetics ISP-BR & DH-2021-C-XI-CH-CK-70- 18. Which one is first order reaction? (A) NH.NO, —N, +2H,O (@) 2HI—=H, +1, (©) 2NO, —+2N0+0, (©) 2N0+0, —+2No0, 49. In the nuclear reaction: JLi1H —-> 2tHe ‘The mass loss is nearly 0.02 amu. Hence the energy released (in units of million Keal /mol) in the process is approx. (A) 425 (©) 220 (©) 120 (0) 50 20. If halflife period is 100 years, average life is nearly (A) 100 years (©) 70 years (©) 144 years (0) 90 years ee Title Led, FITJEE House, ADA, Kalu Sarat, Sarvapriga Vihar, New Delhi 110016, Ph 46106000, Fax 2es1 3942 ‘website: wun fljee.com Chemicalikinetics -ISP-BR & DH-2021-C-XI-CH-CK-71 LEVEL - 11 McQ Single Correct: 1. For any first order reaction following observation is made. If 18 at temperature (T) half life of the reaction is 6930 second & at temperature (T') half life of reaction is 0.693 1 second, then calculate T'/T togk 2 4 6 Os o—a9FeT 2. For a reversible reaction A=" the initial molar concentration of A & B are ‘a’ & 'b' M respectively. If x’ mollitre of A is reacted til the achievement of equilibrium, then x is (k; & ks are rate constants) ka-k,b k,a—k,b (A) 8) ——_ “ k, +k, ® ky -k, ka—k,b katk,b Ok © Fe aA) 3. For reaction A —» B, the rate law expression is—— =! = k[A]"" If intial concentration of (A) is (Alo then, which of the following statement is INCORRECT? (A) The integrated rate expression is k = 2/t (Ao ~ A") (©) The graph of JA vst will be —— aay? (0) The time taken for 75% completion of reaction t,,, = (©) The halflife period ty. = 4. For a hypothetical reaction: X, +¥, —+2XY Follows mechanism as: X, = X+X. (fast) (K, is equilibrium constant) X+Y, = XY+Y. (slow) (K, rate constant) X+Y=ReNy. (fast) (K,, K, are rate constant) The order of overall reaction is (A) 25 1 (©) 32 (0 Chemical kinetics ISP-BR & DH-2021-C-XIL-CH-CK-72- 5. The activity per ml of a solution of radioactive substance is x. How much water be added to 200 mi of this solution so that the activity falls to x20 per ml after 4 half lives? (A) 100 mi (8) 150 mi (C) 80 mi (©) 50mi 6. The high temperature (= 1200 k) decomposition of CH;COOH(g) occurs as follows as per simultaneous 1* order reaction: CH,COOH—*CH, + CO, CH,COOH—SCH,0 +H,0 What would be the percentage of CH, by mole in the product mixture (excluding CHs;COOH)? 50k 100k, wy ke +k, ©) kth, 200k (cy 200k: © rk (O) depends on time 7. A first order reaction is 50% completed in 30 minutes at 27°C & in 10 minutes at 47°C. The energy of activation of the reaction is (A) 43.84 kJ/mol (©) 34.84 kJimol (C) 84.00 kJ/mol (0) 30.00 kJimol 8. The rate equation for the reaction 2A + B —» C is found to be Rate = k(Al[B]. The correct statement is (A) Rate of formation of C is twice the rate of disappearance of A (8) t2is a constant (C) unit of K must be sec" (0) value of K is independent of the initial concentration of A & B 9. Which of the following statement is INCORRECT? (A) Order can be determined experimentally (8) Order of reaction is equal to the sum of powers of concentration terms in differential rate law equation (C) Itis not affected with stoichiometric coefficient of reactant (0) Order cannot be negative 10. The total number of « & f particles emitted in the conversion: U—i Pb (a) 8 @)6 4 2 14. A first order reaction is 50% completed in 20 minutes at 27°C and in § minutes at 47°C. The energy of activation of the reaction is: (A) 43.85 kJ/mol (8) 55.14 ksimol (©) 11.97 ksimol (0) 6.65 kJimol 12. The half-life of B+3C; Alaa cap SE) ap AE] k,[AP the correct relation between ky, kz and ks is (A) Ki=h=ks (8) 2k:=kg=3ky k k, = 73 k, (C) 4ki=k2=3k2 a ©) oF MCQ Multiple Correct Questions: 16. Which of the following statements are correct about half-life period? (A) it is proportional to initial concentration for zeroth order (B) average life = 1.44 x half-life for first order reaction (C) time of 75% reaction is thrice of half-life period in second order reaction (0) 99.9% reaction takes place in 100 minutes for the case when rate constant is 0.0693 min’ 47. 2n+2H > Zn? +H, Halfsife period is independent of concentration of zinc at constant pH. For the constant concentration of Zn, rate becomes 100 times when pH is decreased from 3 to 2. Hence roar lin? — [Zn] | H ) F=K(20)'[H"] ds -P © (2) -Klenr] (C) rate is not affected if concentration of zinc is made four times and that of H” ion is halved (0) rate becomes four times if concentration of H’ ion is doubled at constant Zn concentration Tae Li "Ph 46106000, Fax 26513942 ‘webatte: www fitjee com Chemical kinetics ISP-BR & DH-202 1-C-XIL-CH-CK-74- Pera iif Me Me Me 1 i Which of the following statements are correct? (A) itis unimolecular nucleophilic substitution reaction S,, if | or Ils formed. (8) itis bimolecular nucleophilic substitution reaction S,, if | or Il is formed (©) itis S,, if | and its enantiomer are formed so that mixture is racemic. (0) itis S,,if Wis formed. 49, Rate constant k varies with temperature by equation: logt0K(min'") = 5 ~200°K We can conclude (A) pre-exponential factor Ais 5 (@) is 2000 kcal (©) pre-exponential factor Ais 10° (©) Enis 9.212 kcal 20. A reaction is catalysed by H" ion. In presence of HA, rate constant is 2 « 10° min ‘ and in presence of HB rate constant is 1 x 10° min™', HA and HB both being strong acids, we may conclude (A) equilibrium constant is 2 (8) HAs stronger than HB (C) relative strength of HA and HB is 2 (0) HA is weaker than HB and relative strength is 0.5 21. For the reaction, 3X——>Y +22, the value of the rate constant when rate is expressed in is 0.036 L mors" So, when rate is expressed in terms of “YI then (A) rate constant =12x10°Lmot's (®) rate constant = 7.4x10°Lmol's (C) molecularity = 3 (0) molecularity = 2 22. In Arrhenius’ equation, k = Ae“*/"" A may be regarded as the (A) rate constant at very high temperature (8) maximum rate constant (C) rate constant at negligible activation energy (0) rate constant at very low temperature 23. The rate of a reaction (A) always increases with increasing concentration of the reactants (8) depends upon the temperature (C) is same for different reactions at same temperature (0) is increased by adding a catalyst 24, ky 28 5 ky = 105" ax ko 2« 102s" BY3p 75 104s For the given reaction, which of the following is/are correct? (A) (8) (C] []=2 1 3 ©) (8) (C] ()=2 2 15 (©) ty, A = 198 seconds (©) (Al= [Al + 218] + (C] + 310] —— Vaile Lid, FITUBE House, 29-4, Kalu Sarat, Sarvapriga Vihar, New Delht 110016, Ph 46106000, Fax 20813942 ‘website: wusw fltfec.com eR sini alitinetics ISP-BR & DH-2021-C-XI-CH-CK:75 25. Two stereoisomers of D-glucose are a-D-glucose and p-D-glucose having specific rotations of +112° and +19° respectively. When 3.42 g of a-D-glucose is dissolved in 2 litre of HzO after certain time, the specific rotation of the aqueous solution of «-D-glucose comes to a constant value of +52.7° Regarding this, which are the correct statements given below? (A) In aqueous solution, «-D-glucose converts totally into i-D-glucose. (B) In aqueous solution, a-D-glucose partially gets converted to p-D-glucose and an equilibrium is maintained between them. (©) The equilibrium constant is 1.78. (D) There is no question of equilibrium constant. Assertion Reason Type Questions: (A) If both assertion and reason are correct and reason is the correct explanation of the assertion, (B) If both assertion and reason are correct but reason is not the correct explanation of the assertion, (C) If assertion is correct but reason is incorrect, (0) If both assertion and reason are correct. 26. [Assertion]: K xe" the Arrhenius equation represents the dependence of rate constant with temperature. [Reason]: Plot of log K against t is linear and the activation energy can be calculated with this plot. 27. (Assertion]: A catalyst enhances the rate of a reaction. [Reason]: The energy of activation of the reaction is lowered in presence of a catalyst. 28, [Assertion [Reason] Order of reaction can never be fractional for an elementary reaction Elementary reactions take place by one step mechanism Comprehension Based Questions: Comprehension - 1 A reaction is said to be of zero order if its rate is independent of the concentration of the reactants, i.e. the rate is proportional to the zeroth power of the concentration of the reactants. For a reaction, A—P, to be of zero order, & _kgap dt mK at Some photochemical reactions and a few heterogeneous reactions are zero order reactions. Such reactions are not common, Read the above paragraph carefully and answer the questions given below it: 29. For reaction of zero order, 2A——>3B with initial concentration of A as 2m/ and rate constant 4m/¢/hr. The concentration of A after 30 min, (A) Zero ©) 1.5 m/e (©) 1.25M (0) 0.25M wee ‘Sarvapriva Vihar, New Deiht 110016, Ph #6100000, Fax 26513903 ‘webat wow fltfec.com Chemical kinetics ISP-BR & DH-2021-C-XIL-CH-CK-76: 30. For above, zero order reaction, time of completion will be, (A) 15 min. (©) 7.5 min. (C) 30 min, (D) 60 min, Comprehension - It ‘A catalyst increases the rate of chemical reaction by providing an alternative reaction pathway which lower activation energy than the reaction pathway in its absence. A catalyst is not consumed and therefore does not appear in the chemical equation for the reaction. A homogeneous catalyst is in the same phase as the reactants while t a heterogeneous catalyst is in a different phase. The complex [Co(NH;)-F]”’ reacts with water according to the equation; [Co(NH,).F]** +H,O—[Co(NH, ),(H,O +F)}°* and Rate = K [complex]* [H’]” The reaction is acid catalysed i. [H'] does not change during the reaction Thus rate = K'[eomplex}* where, K’ =(H"}® Complex Wy Ta(hr) | Ty (hr) 0.1 0.01 1 2 02 0.02 05 1 Read the above paragraph carefully and answer the questions given below it: 31. From the above data, value of a is: wt @)2 1 (C) 0 (0) = ©) > 32. Overall order of reaction is: at @2 1 Cc) 0 0) 1 © > Comprehension - Ill ‘Suppose 50 bacteria are placed in a flask containing nutrients for the bacteria so that they can multiply. A study at 35°C gave the following results: Time (minutes) 0 15 30 45 60 Number of bacteria 100 200 400 800 1600 Then 33. This multiplication of bacteria follows a (A) zero order reaction ®) 1* order reaction (©) 2” order reaction (0) 3° order reaction 34. The rate constant for the reaction is (A) 0.0462 min” (8) 0.462 min” (C) 4.62 min* (0) 46.2 min Ted., FUTUBE House, 29°, Ratu Sarat, Sarvapriya Vihar, New Delhi 110016, Ph #6106000, Fax 26513942 ‘webatte: wow ftjee.com Chemical kinetics erica Kine ties isp BR & DH-2021-C.XI-CH-CK-77 Matrix Match Type Questions: 36. Match List ~ | with List -I ate z Toe List (P) | Probability of survival of a radioactive nucleus | (1) | No(2)” for one mean life _ (Q) | Amount remaining after n-half lives (2) | ext! - —| ® &, Ry = rate at time ts, Re = rate at time ty @ (3) i 2. | Amount left on 11” day, starting with 1 mole of A 1 9) | (ty = 10 days) Ols : P Q eR s @M2 1 4 3 @®3 2 1 4 (C)4 1 2 3 (0)4 3 2 1 36. Match Column — | with Column - Il and choose the correct answer from the code given below Column =1 - Z Column = TA) | Halflife period of first order reaction | (p) ©) | Molecular concentration @ (C)_| Temperature coefficient ()_| active mass - (0) | Half life period of zero order reaction | (s) | a 2K T_ @_| Independent of initial concentration Table-1 [ ‘Column-1 ‘Column-2 ‘Column-3 {Order of reaction) (Half Life) (Concentration) @_[ Zero order [tz = [Agr e) = (ii)_| First order i) [tuo & fg Q) (ut) | Second order il) | tre % (Aol? 3) (IV) | Third ord Ww) (S) ! 1 2Kt | iird order (iv) | tre % [Ao] ( far ia far [acl 37. The only correct combination is (MH) R) ® Owe) ©) WY) GS) 38. Which one is only incorrect combination is (A) (W) dil) P) C) () MR) 39. The only correct combination is (A) (i) @(R) (C) (tl) Gi) (Q) Vaile Led, FITUBE. (O) (H) (iv) (S) (8) (Hi) (Q) 0) vy) Q) 6) (MOR ©) () @) Q) "Ph 46106000, Fax 26519942 Chemical kinetics ISP-BR & DH-2021-C-XIL-CH-CK-78- Single Integer Type Questions/Decimal Based Questions: 40. For the reaction, 2N,O, —+4NO, +0, rate and rate constants are 1.02 x 10 M sec ' and 3.4 10° sec’ respectively, then concentration of NOs (in molarity) at that time will be. 41. In a gaseous phase reaction: (following first order reaction) A; (g) —>B(a)+C(a), the increase in pressure from 100 mm to 120 mm is noticed in 5 minutes. The rate constant in hr" is. 42. In gases reaction important for the understanding of upper atmosphere H20 and O reacts bimotecularly to form two OH radicals. aH for this reaction is 72.32 kJ at 500 K and E, is 77.5 kJ mot’, then E, (in kImor) for the bimolecular recombination of two OH radicals to form H,0 and Os, 43. A fresh radioactive mixture containing short lives species A and B. Both emitting «particles initially of 8000 a-particles per minute. 20 minutes later, they emits at the rate of 3500 corparticles per minute. If the half-lives of the species A and B are 10 minutes and 500 hours respectively, then the ratio of activities AB in the initial modure was x 1. What is the value x? ‘Wile Led, FITUEE House, 294, Kalu Sarat, Sarvapriga Vihar, New DelN 110016, Ph 46106000, Fax 26310942 ‘webolte: wawfitjee.com Chemical kineti¢s isp pe a pH.2021.0-XICH.CK-79 ANSWERS TO CHAPTER PRACTICE PROBLEMS (CPP) STS Subjective: a 1. 600/4.606 K 2, 378.74K 3 5 4 6 100.82 ———————— Objective: 1 c 2. c 3 c 4 c 5 c 6. AB 7 A.B,D 8 B,C,D 9 2 10. 0 ee Led, FITUEE ‘webatte: wow fltjee.com Chemical kinetics ISP-BR & DH-2021-C-XIL-CH-CK-80- ANSWERS TO ASSIGNMENT PROBLEMS Subjective: LEVEL 1. Interms of concentrations, units of rate = mole lit ‘mit Rate ___ mole lit" min-* (HOCH) (mote it") In terms of pressure, unit of rate = bar min” bar min” nm pane bar bar 2 o S-K(aley (i) rate = Kab? if Blis tripled, Rate = Ka(3b)’ = 9Kab’ = 9 times if both [A] and (B] are doubled, Rate = K(2a)(2b)* = 8Kab’ = 8 times 3. Rate depends on the slow step. Unit of K = L'? mole ?min"* Unit of K = min” Rate =K[NOs][NO]..() for first reaction if equilibrium constant is Ke - NO) NO}[O2] [No,] =k. {No][0.] Putting the value of [NOs] in equation () Rate = K.K.{NO]"[O2] KKK Rate = K'[NO}"[O2) The overall order of reaction is 3. 4. Orderis 1 5. If E,=0, then according to Arhenius equation, K= Ae®'"" = Ae® =A ie, rate constant = collision frequency. This means every collision results into a chemical reaction which can not be true. Hence E, 20. 6. A bimolecular reaction may become kinetically of first order if one of the reactant is present in ences 16. (a) Rate equation for the reaction: r =K[C1,O}° Letnon ate verien e=r[ 22] =} (b) In order to have the rate = 2r, let the concentration of C1,O be x. so 2K ) Taiiee Ltd, PITJBE House, 2-4, Kalu Sarai, Sarvapriya Vihar, New Delht 110016, Ph 46106000, Fax 20519942 ‘webatte: wunw fltjee.com Chemical kinetics -ISP-BR & DH-202 1-C-XI-CH-CK-81 we known that r=K([CLO]? ii) Dividing equation (i) and (ji) 2 2 © [cof (c1,oF 2{C1,0]° orx= V2[Cl,0] (©) New rate = K[3Cl,]’ = 9K[CI,O]’ = 9r nine times of original rate, 17. According to this theory, the reaction takes places as a result of the collision between the reactant molecules. The number of collisions per unit volume per unit time is known as collision frequency Zs, But all collisions are not effective. The collision which actually converts the reactants into products are called effective collisions. These collisions takes the ‘molecules to the top of the energy barrier and finally results in the formation of products. There are two conditions for the effective collision: () Energy barrier: The reactant molecules must posses the minimum energy known as threshold energy before they could react and form products. (i) Orientation barrier: The reactant molecules must be property oriented in order to have an effective collision The rate constant is given by then k=PZge Where Zas = collision frequency P = orientation factor and e*%’*" = fraction of total colision which are effective. 18. The symbol °,e represents the electron in or from an atomic orbital. The symbol °,3 represents an electron, that although physically identical to any other electron, comes from a nucleus and not form an atomic orbital The emission of °f) involves the conversion of a neutron into a proton. 19. () 410424 (i) $Mn +n Mn43 y 20. (i) Mass defect is the difference between the actual mass of an isotope of an element and the sum of the masses of protons, neutrons and electrons present in it. (i) Binding energy is the energy required to hold the nucleus together. Binding energy of a nucleus is generally quoted as energy in million volts (MeV) per nucleons. The binding energy per nucleon is a measure of the stability of the nucleus. The greater the binding energy per nucleon, the more stable is the nucleons, "or (mol L“')"'s"” Equate this with general expression 21. The units of rate constant is L mot” of (mol L™")""s"! (motte) 's” -norn= The order of reaction = 2 mot L')"s* 22. ())_2NH, (g)—* oN, (9) + 3H, (9) = INH)” (i) NH.NO, (s)—n, (9) +H,0(¢) = K[NH«NO)] ——— eee —————————— WNC Led, PITUBE House, 29-4, Kalu Sarat, Sarvapriya Vihar, New Delhi 110016, Ph 46106000, Fax 26519942 ‘webatte: war fgjee.com Chemical kinetics ISP-BR & DH-2021-C-XI-CH-CK-82- 23. No. of hatf ite = 240 60 A Amount left [A] = {Ab = 0.0625 of (A]o= 6.25 % 24, Emission of particle produces isobars B'U—>,) No+°, B'Uand 2Np are isobars LEVEL -1 1. 1,944«10?mole/lit sec 2. [0.5 mole tit *hr-',0.25 mote lit‘hr-*] 3. [53.59 KJmol"] 4 (K=3.19«107Lmor's") 8. (64x10") 6. 23 seconds 7. Rate =K(H, [NO], K =1.45.10°7tmor’s" 9K =2.232x10? min", 2.12 min 10. 99.7 sec 41. (a) 7.2 mol L"'min”; (b) 7.2 mol L 12. (i) 7.2M; (i) Think yourself 13. (i) 36 minutes; (ji) 108 minutes 14, Zero order 15, 349.1 K LEVEL ~11 4. tia = 128.3 hrs, 5.27% 2 84x10%s" 3. 12081.7 Pa 4. 10.924 Keal/mole 5. 0.03265 mi 6 K= 4.410" min 7, 0.0102 min'* 8 8 days 9. 100% 10. Original quantity of the isotope (Nc) = 1 gm. Final quantity of the isotope, N = 0.25 g we soon tat (2 ime Led, PITUBE eBemicsiieinetios -ISP-BR & DH-202 1-C-XII-CH-CK-83 Time taken = T= 1 x tya= 2 x 140 = 280 days 11. k= 258g FITUEE House, 294, Kalu Saral rapriva Vihar, New Delht T1001 ‘website: wow ftjer.com "Ph 46106000, Fax 26519962 Chemical kinetics ISP-BR & DH-2021-C-XIL-CH-CK-84- Objective: LEVEL -1 1 A 2 c 3 c 4 A 5. c 6 ° 7 8 8 8 9 B 10. ¢ uw 8 12° ~O 3. 14.0«A 15 16D 7 OA 18 OA 19. A 20. ¢ LEVEL -I 1 A 2 A 3 c 4 c 5 o 6. A 7 A 8 D 9 D 10. A 1B 12. «=D 3° ¢ 14. € 15D 16. A,B,C,D 17 B,C,D 18 CD 19. ¢,0 20. B,C 2.0 AC 22, ABC 23. B,D 24. B,C 25. B,C 26. OA aA 2. B 2. =A 30. «¢ 31 OA 32.0«=w@ 3. B 34. OA 35. 36. (A) > (4, t); (B) > (95 (C) > (p); (D) - (s) a7 38. 0O«B 30. A 40. 3 41,267 42, 8.18 43. 3 "Hilt Led, PITUBE House, 25°, Kalu Sarat, Sarvapriya Vihar, New DelN 110016, Ph 46106000, Fax 26513942 ‘website: war fljee.com WHEN IT COMES TO YOUR CAREER, _YOU CANNOT AFFORD TO TAKE CHANCES | CHOOSE THE BEST

You might also like